Mechanische Systeme und Koordinatensysteme

BLOG: Quantenwelt

Gedanken eines Experimentalphysikers
Quantenwelt

Der Duden definiert System in den Naturwissenschaften als “Gesamtheit von Objekten, die sich in einem ganzheitlichen Zusammenhang befinden und durch die Wechselbeziehungen untereinander gegenüber ihrer Umgebung abzugrenzen sind”. Diese Definition ist richtig, führt aber in Gedankenexperimenten zur Relativitätstheorie gern zu Missverständnissen.

Typische Gedankenexperimente beinhalten zum Beispiel einen Zwilling in einer Rakete und einen auf der Erde, oder einen Zug und einen Bahnsteig, oder zwei Raketen. Manchmal auch Panzer, Gruben, Autos und Garagen. In Raketen, Zug und Bahnsteig befinden sich eine oder mehrere Uhren, ein paar Beobachter, es fliegen Blitze umher. Die Garage hat Türen und irgendeinen Mechanismus, diese zu schließen. All diese Konstellationen können als Systeme betrachtet werden.

Mechanische Systeme

In meinem Artikel zu Luhmanns Systemtheorie beschrieb ich Systeme als Teile der Welt, bei denen deutlich mehr Interaktionen innerhalb des Systems als zwischen den Systemen stattfinden. Raketen im Weltraum sind dafür das perfekte Beispiel: Innerhalb der Rakete gibt es Luft, sie ist durch Wände begrenzt, alles was in ihrem Inneren passiert beeinflusst in erster Linie nur die Rakete selbst. Außerhalb der Rakete ist Vakuum und eisige Kälte. Die Konstruktion der Rakete soll Austausch mit der Umgebung im Wesentlichen unterbinden. Auch Bahnsteig und Zug aus Einsteins Original-Gedankenexperiment sind im Wesentlichen voneinander getrennte Systeme. Der Zug führt, ähnlich wie die Rakete, seine eigene Atmosphäre mit, damit den Fahrgästen nicht der Kaffee aus den Bechern weht.

Schreiben wir nun, dass das Licht im Raketensystem mit Lichtgeschwindigkeit unterwegs ist, so kann leicht der Eindruck entstehen, es sei das innere der Rakete gemeint. Bei durchtreten des Fensters gehe das Licht dann ins Weltraumsystem über und dann irgendwann in das Erdsystem oder ein anderes Raketensystem.

Inertialsysteme sind globale Koordinatensysteme

Tatsächlich ist mit dem Raketensystem aber nicht das mechanische System Rakete gemeint. Das Raketensystem ist das Ruhesystem der Rakete. Und ein Ruhesystem ist ein Koordinatensystem. Ein Inertialsystem. Die Rakete kann als Referenz genommen werden, um ein Koordinatensystem aufzuspannen, in dem die physikalischen Gesetze in ihrer einfachsten Form gelten und das die ganze Welt umspannt. Es befindet sich also nicht nur alles im Ruhesystem der Rakete, das sich in oder in unmittelbarere Nähe der Rakete befindet. Die andere Rakete, die Erde oder was auch immer Teil des Gedankenexperiments ist befindet sich ebenfalls im Ruhesystem der Rakete.1

Systemwechsel

Weil es sich beim Raketensystem um ein globales Koordinatensystem handelt, kann kein Objekt des Gedankenexperiments jemals dieses System verlassen. Es erstreckt sich ja über die ganze Welt. Objekte können in einem Koordinatensystem betrachtet ruhen, sie können sich geradlinig mit konstanter Geschwindigkeit bewegen und sie können sogar beschleunigt sein.

Entscheidend ist, dass für alle Objekte in jedem beliebigen Ruhesystem dieselben Gesetze gelten. Zeit vergeht für bewegte Objekte langsamer, ihre Länge ist relativ zu dem, was sie selber messen, verkürzt. Für beschleunigte Objekte kommen die klassischen Trägheitseffekte hinzu: Kräftefrei bewegt sich ein Objekt stets geradlinig mit konstanter Geschwindigkeit. Jede Abweichung von geradliniger, gleichförmiger Bewegung muss also von äußeren Kräften oder inneren Spannungen begleitet sein. Relativistisch kommen weitere Effekte hinzu.

Ein Wechsel des Koordinatensystem ist also eigentlich nicht nötig, jede Situation kann vollständig in jedem beliebigen Ruhesystem beschrieben werden. Der Systemwechsel ist im Wesentlichen eine Umbenennung von Koordinaten. Dabei kann dich die gesamte Situation scheinbar ändern: Objekte, die vorher in Ruhe waren, bewegen sich. Zeitläufe erscheinen anders, Gleichzeitigkeiten und Abstände ändern sich. Bei einem Wechsel von kartesischen Koordinaten zu Kugelkoordinaten ändert sich sogar die Vorschrift, nach der Abstände berechnet werden.

Messwerte und Koinzidenzen ändern sich nicht

Messungen finden in der klassischen Physik2 immer lokal statt. Zwei Uhren begegnen einander und werden verglichen, eine Uhr wird mit einer vorbeifliegenden abgeglichen, ein LIchtstrahl kommt am Detektor an und löst ein Ereignis  aus, eine Rakete kommt irgendwo an und wird beschleunigt oder abgebremst.

Da Koordinaten nichts anderes sind als die Kennzeichnung von Ort und Zeit jedes lokalen Ereignisses mit vier Werten, kann ein Wechsel des Koordinatensystems nichts an den Messungen und Koinzidenzen ändern. Was in einem Koordinatensystem zur gleichen Zeit am selben Ort geschieht, geschieht auch in allen anderen Koordinatensystemen gleichzeitig und lokal.

Was Gedankenexperimente mit Koordinatenwechsel zeigen sollen, ist nicht, dass Koordinatenwechsel die Messwerte und Vorgänge unverändert lassen. Das ist trivial. Entscheidend ist die Frage, welche Formen von Koordinatenwechsel die physikalischen Gesetze unverändert lassen. Drehungen und Verschiebungen von Koordinatensystemen gehören in jedem Fall dazu. Galileo hat gezeigt, dass auch zueinander gleichförmig bewegte Koordinatensysteme die Physik unverändert lassen. Einstein zeigte dann, dass das nur streng gilt, wenn Zeit- und Längenskala entsprechend angepasst werden.

Anmerkungen:
1. Streng genommen gibt es beliebig viele Ruhesysteme der Rakete. Hat man nämlich eines konstruiert, so ist jedes zu diesem System verschobene oder verdrehte Koordinatensystem ebenfalls ein Ruhesystem der Rakete. Dazu kommt, dass ein Koordinatensystem nicht nur als das bekannte rechtwinklige kartesische Koordinatensystem mit den Raum Koordinaten X, Y und Z dargestellt werden kann. Gebräuchlich sind auch Zylinder-Koordinaten mit einer Zylinderachse, einem Radius und einem Winkel oder Kugelkoordinaten mit einem Radius und zwei Winkeln.
2. und die Spezielle Relativitätstheorie ist eine klassische Theorie
Avatar-Foto

Veröffentlicht von

www.quantenwelt.de/

Joachim Schulz ist Gruppenleiter für Probenumgebung an der European XFEL GmbH in Schenefeld bei Hamburg. Seine wissenschaftliche Laufbahn begann in der Quantenoptik, in der er die Wechselwirkung einzelner Atome mit Laserfeldern untersucht hat. Sie führte ihn unter anderem zur Atomphysik mit Synchrotronstrahlung und Clusterphysik mit Freie-Elektronen Lasern. Vier Jahre hat er am Centre for Free-Electron Laser Science (CFEL) in Hamburg Experimente zur kohärenten Röntgenbeugung an Biomolekülen geplant, aufgebaut und durchgeführt. In seiner Freizeit schreibt er zum Beispiel hier im Blog oder an seiner Homepage "Joachims Quantenwelt".

100 Kommentare

  1. Das Schöne an der Relativitätstheorie ist ja dass sie seit 100 Jahren mehr oder weniger kontinuierlich erklärt wird.
    Für Beschäftigung der Autoren und der Leser sorgt das allemal.
    Ist doch auch etwas.

  2. @Markweger 26. Juni 2018 @ 22:08

    Und Newton wird seit 300 Jahren erklärt. Nun dafür sorgen schon Leute, die behaupten, dass diese Theorie nicht stimmt. Eigentlich können diese Leute dankbar sein, dass es das Internet gibt. Denn sonst würde man sie ja nicht bemerken.

    Zum Artikel:
    Das ist sicher ein wichtiger Punkt, dass die Bezugssysteme zwar an ein Objekt “gebunden” sind, aber in ihnen auch andere zu diesem Objekt bewegte Objekte umfasst. In der Teilchenphysik sind ja die Objekte nicht mal mit dem Mikroskop zu erkennen. Trotzdem haben sie ein Bezugssystem, dass auch das Labor umfasst.

  3. Na ja, Newton ist im Prinzip schon verständlich, sofern einem Mechanik nicht völlig fremd ist. Newtons Mechanik ist logisch und plausibel.
    An der Relativitätstheorie ist weder irgend etwas logisch noch ist irgend etwas plausibel.

  4. @Markweger 27. Juni 2018 @ 09:54

    Nun denn. Welche Einwände haben Sie gegen die Erklärung des Dopplereffektes im vorigen Artikel von Herrn Schulz? Dessen Titel war “Mit dem Dopplereffekt Relativität durchschauen”. Für mich war die Erklärung plausibel.

  5. Es gibt in der Öffentlichkeit allein schon viele Missverständnisse was die Relativitätstheorie angeht, indem die Leute falsch einschätzen, wann sie zu berücksichtigen ist und wann nicht.

    Ein Gymnasiumskollege meinte beispielsweise beim (hochpräzisen) Turbinenbau müsse wohl auch die Relativitätsheorie berücksichtigt werden – wegen den hohen Geschwindigkeiten. Es ist allerdings sehr relativ, was eine hohe Geschwindigkeit ist.

    Überraschen wird vielleicht auch einige, dass bei Universumssimulationen wie Illustris, welche die Entstehung des gesamten Universums simulieren und die damit auch die Schwerkraft simulieren müssen, die Allgemeine Relativitätstheorie nicht benutzt wird, sondern nur die Netwon’schen Formeln für die Gravitation. Grund: Die allgemeine Relativitätstheorie weicht in den numerischen Resultaten nur in Extrembereichen des Universums wie den schwarzen Löchern und ihrer unmittelbaren Umgebung so stark von der Newtonschen Formel ab, dass sie berücksichtigt werden muss. Auf die Bildung der grossen Strukturen im Weltraum hat sie nur indirekt Einfluss (über die schwarzen Löcher und ihre Funktion bei der Strukturentstehung).

    Andererseits gibt es auch viele Leute, die meinen, die Relativitätstheorie spiele für unseren Alltag überhaupt keine Rolle. Auch das stimmt nicht. Im Alltag begnet sie uns etwa bei folgenden Phänomenen
    – Orts-Zeitbestimmung mit GPS-Satelliten (wegen der nötigen hohen Präzision)
    – Beschuss der Erdoberläche durch etwa 100 Myonen pro Quadratmeter und Sekunde, die in der Erdatmosphäre in 10 km Höhe entstehen, wenn kosmsiche Strahlung auftritt. Die Hälfte der Myonen ist nach 1,523 µ-Sekunden (Millionstel Sekunden) zerfallen, womit nicht erklärt ist warum die Myonen überhaupt bis zur Erdoberfläche vorstossen ohne vorher zu erfallen, doch die hohe Geschwindigkeit der in der oberen Atmosphäre erzeugten Myonen verlängert über die Zeitdilatation ihre Lebensdauer so stark, dass sie bis zur Erdoberfläche vorstossen. Diese Myonen, die uns erreichen erhöhen die (radioaktive) Hintergrundsstrahlung, erlauben aber auch Techniken wie die Myonentomographie, beispielsweise in Form der Durchleutung der Pyramiden um in den Pyramiden bis jetzt unentdeckte Hohlräume zu finden.
    – Jeder elektrische Strom wird von einem Magnetfeld umgeben und dieses Magnetfeld entsteht durch Längenkontraktion der bewegten Elektronen relativ zu den unbewegten positiv geladenen Atomkernen des Metalls, was zu einem zusätzlichen elektrischen Feld führt, welches wir aufgrund seiner speziellen räumlichen Anordnung Magnetfeld nennen.

  6. Ich denke tatsächlich darüber nach, etwas mehr Grundlagen der Newtonschen Mechanik zu erklären. Die Diskussionen, vor allem mit so genannten Kritikern der Relativitätstheorie, zeigen nämlich, dass bereits dort die falschen Vorstellungen vorherrschen. Vielen Laien ist klar. dass sie die SRT nicht durchschauen. Wo die Newtonsche Physik überraschend ist, ist vielleicht weniger klar. Das Galileis Relativitätsprinzip ist ungefähr genau so kontra-intuitiv wie Einsteins.

  7. Kann man sagen (um System und Bezugssystem zu unterscheiden) —– da (Zitat) System in den Naturwissenschaften die “Gesamtheit von Objekten, die sich in einem ganzheitlichen Zusammenhang befinden und durch die Wechselbeziehungen untereinander gegenüber ihrer Umgebung abzugrenzen sind“ ist —– dass ein System aus einzelnen Bezugssystemen besteht und als Ganzes aber wiederum ebenfalls ein Bezugssystem ist?

    Gemäß https://www.spektrum.de/lexikon/physik/ruhesystem/12635 ist ein R. ** dasjenige Inertialsystem, in welchem ein sich beliebig bewegter Körper im betrachteten Moment ruht. **

    Für mich (Laie) ist Ruhesystem ein zwiespältiger Begriff. Einfach assoziiert möchte ich meinen, dass ein R. ein System ist, das sich nicht bewegt. Also z.B. wenn ich auf der Erde stillstehe. Dann ist das (von mir aus gesehen, auf mich bezogen) ein/mein Ruhe- bzw. Bezugssystem. Obwohl ich mich ja mit der Erde (ihrer Drehung um sich selber) bewege. Und diese wiederum (selbst wenn sie nicht rotierte) ja um die Sonne kreist. Und selbst wenn das nicht der Fall wäre, unser Sonnensystem ja um das Zentrum der Milchstraße. Und wenn das nicht wäre (die Milchstraße – in ihrem Bezugssystem – bewegungslos ruhte), sie sich ja, sagt man, auf den Großen Attraktor zubewegt. Und der (selbst wenn auch er ruhte) sich ja innerhalb/mit der Expansion des Universums bewegt. Man kann auch noch anmerken, dass sich (wenn ich auf der Erde stillstehe) in mir ja meine Atome bewegen (nicht stillstehen). Das ist dann aber ein anderes Bezugssystem (eine andere Sichtweise) – obwohl es Teil meines “Ruhe”-/Bezugssystems ist. “Oder?”

    Unter Inertialsystem schreiben Sie (bei http://www.xn--relativittsprinzip-ttb.info/inertialsystem.html): ** [..] Insbesondere würde sich ein kräftefreier Körper in diesen Koordinaten gleichmässig auf einer geraden Linie bewegen [..] **. Oder, hier im blogpost: ** Kräftefrei bewegt sich ein Objekt stets geradlinig mit konstanter Geschwindigkeit. ** Diese gradlinige konstante Bewegung hat aber ja auch Mal eine (“Kraft” als) Ursache gehabt (“von nichts kommt nichts“).

    Es gibt offenbar kein wirkliches Ruhesystem (“panta rhei“) – keinen universellen Bezugspunkt/-begriff, den man zum Maßstab (zur Orientierung/Richtlinie) nehmen kann. “Alles” ist relativ zu allem anderem/untereinander. Außer ich nehme, sozusagen, mein persönliches/lokales Bezugssystem zum Maßstab. Oder die Erde, das Sonnensystem, die Milchstraße usw. (jeweils inkl. mir).

    Frage (Zitat): Messungen finden in der klassischen Physik immer lokal statt. Was ist (in diesem Zusammenhang) “unklassische“ Physik?

    Und (Zitat): Was in einem Koordinatensystem zur gleichen Zeit am selben Ort geschieht, geschieht auch in allen anderen Koordinatensystemen gleichzeitig und lokal. Das meint nicht (oder?), dass identische Ereignisse stattfinden, sondern dass wenn hier etwas passiert woanders gleichzeitig auch etwas passiert.

    Damit das nicht zu “philosophisch” rüberkommt, folgendes: 1 Lichtjahr entspricht der Stromrechnung für 12 Monate …

  8. @Axel Krüger 27. Juni 2018 @ 11:39

    Frage (Zitat): Messungen finden in der klassischen Physik immer lokal statt. Was ist (in diesem Zusammenhang) “unklassische“ Physik?

    Dies ist die Quantenphysik. Da hat aber Herr Schulz auch eiiges geschrieben. Besonders interessant sind dabei “verschränkte Teilchen”.

    Und (Zitat): Was in einem Koordinatensystem zur gleichen Zeit am selben Ort geschieht, geschieht auch in allen anderen Koordinatensystemen gleichzeitig und lokal. Das meint nicht (oder?), dass identische Ereignisse stattfinden, sondern dass wenn hier etwas passiert woanders gleichzeitig auch etwas passiert.

    Nun wenn an einem Ort in einem Koordinatensystem zwei Ereignisse gleichzeitig passieren, dann passieren sie im anderen auch zur gleichen Zeit. Ein Beispiel dazu:

    Angenommen ein Stab mit zwei Spiegeln und einer Lichtquelle in der Mitte bewegt sich gegenüber einem Messystem. Wenn diese Lichtquelle ein Blitz aussendet, dann treffen sie im System des Stabes gleichzeitig auf die Speiegel und im Messystem nicht gleichzeitig. die reflektierten Lichimpulse treffen aber in beiden System gleichzeitig einen Empfänger in der Mitte.

  9. @Axel Krüger
    Um es zusammenzufassen:
    Ein mechanisches System ist ein System aus physikalischen Objekten, die sich irgendwie in der Welt bewegen und miteinander Wechselwirken.
    Ein Koordinatensystem ist ein System aus Zahlen und Einheiten, die man nutzt um Orte und Zeiten in einem physikalischen (Gedanken-)Experiment eindeutig zu kennzeichnen.

    Physiker verstehen unter einem Bezugssystem und vor allen unter einem Ruhessystem fast immer ein Koordinatensystem.
    Das Missverständnis entsteht, wenn sich der Laie unter einem Bezugssystem ein mechanisches System vorstellt.

  10. Zitat Axel Krüger: „Kräftefrei bewegt sich ein Objekt stets geradlinig mit konstanter Geschwindigkeit. „ Diese gradlinige konstante Bewegung hat aber ja auch Mal eine (“Kraft” als) Ursache gehabt (“von nichts kommt nichts“).
    Es gibt offenbar kein wirkliches Ruhesystem (“panta rhei“) – keinen universellen Bezugspunkt/-begriff, den man zum Maßstab (zur Orientierung/Richtlinie) nehmen kann. “Alles” ist relativ zu allem anderem/untereinander.”

    .

    So ist es: Es gibt im ganzen Universum keinen Zustand der absoluten Ruhe, alle Objekte bewegen sich relativ zu allem andrem/untereinander, alle.

    Und gerade daran liegt der Grundfehler Einsteins, der es nicht erkannt hat, wobei er das Relativitätsprinzip Galileis, das er in seinem 1. Postulat für seine SRT zugrunde gelegt hat, völlig falsch interpretiert hat!

    Galilei hat nämlich ausgesagt, dass es bei zwei relativ zueinander bewegten Objekten nicht möglich ist festzustellen, welches der beiden Objekte ruht und welches sich bewegt. Diese Aussage ist auch ganz einfach von Laien zu verstehen und kann gelegentlich in der Empirie erlebt werden: Wenn zum Beispiel zwei Zügen parallel auf zwei Gleise stehen, und ein der Züge fängt an zu fahren, oder auch beiden, können die Passagiere nicht feststellen, welche der beide Züge fährt und welcher ruht, oder ob beiden fahren. Wer hat sich in solchen Situationen nicht gefragt: „Fahren wir, oder fährt der andere Zug?

    Aus diesem Umstand kann man rein kinematisch (also ohne Berücksichtigung von Kräften) bei zwei zueinander bewegten Objekten wahlweise bzw. abwechselnd ein der Objekte als ruhend und das andere als bewegt festlegen, das ist rein kinematisch völlig egal.

    Galilei hat jedoch nie behauptet, dass bei zwei relativ zueinander bewegten Objekten ein Objekt zwangsläufig ruht und das andere sich bewegt! Galilei wusste natürlich, dass bei zwei relativ zueinander bewegten Objekten beide Objekte auch mit eigener Geschwindigkeit sich bewegen können. Wenn zum Beispiel ein Objekt sich mit eigener Geschwindigkeit v entgegen eines Lichtstrahls bewegt, ist dann selbstverständlich die Relativgeschwindigkeit zwischen den beiden nach der Geschwindigkeitsaddition Galileis c + v.

    Einstein hat aber das Relativitätsprinzip Galileis nicht verstanden bzw. völlig verunstaltet: Er hat für seiner SRT zugrunde gelegt, dass zwischen zwei relativ zueinander bewegten Objekten ein der Objekte zwangsläufig ruhen muss, also die Geschwindigkeit v=0 haben muss! Deshalb lautet die Geschwindigkeitsaddition Einsteins mit einem Lichtstrahl c+0=c. Die absolute Konstanz der Lichtgeschwindigkeit ist nichts Anderes als eine Addition mit 0. Dies bedeutet physikalisch, dass für Einstein kein einziges Objekt im Universum eine eigene Geschwindigkeit hat, alle haben die Geschwindigkeit v=0, alle Objekte im Universum ruhen absolut, nur das Licht bewegt sich im Universum. Ein völlig abstruses Weltbild… 🙁

  11. @Lopez

    Einstein hat aber das Relativitätsprinzip Galileis nicht verstanden bzw. völlig verunstaltet: Er hat für seiner SRT zugrunde gelegt, dass zwischen zwei relativ zueinander bewegten Objekten ein der Objekte zwangsläufig ruhen muss, also die Geschwindigkeit v=0 haben muss! Deshalb lautet die Geschwindigkeitsaddition Einsteins mit einem Lichtstrahl c+0=c.

    Nun kommen Sie wieder mit demselben Unsinn daher. Es ist sicherlich so, dass nach Einstein ein Objekt in einem Inertialsystem ruht. Dies gilt aber auch bei Galilei. Und in Ihrer Widerlegung des Additionstheorems der Geschwindigkeiten setze Sie auf die linke Seite c+v und auf der rechten Seite die Formel für das Additionstheorem mit v2=c. Dann kommt man natürlich auf v=0. Mal sehen, was Herr Schulz dazu sagt. Übrigens können Sie keine Hilfe von Chryptic erwarten. Der hat hier nichts zu sagen.

  12. Zitat Rudi Knoth: „Nun kommen Sie wieder mit demselben Unsinn daher. Es ist sicherlich so, dass nach Einstein ein Objekt in einem Inertialsystem ruht. Dies gilt aber auch bei Galilei. Und in Ihrer Widerlegung des Additionstheorems der Geschwindigkeiten setze Sie auf die linke Seite c+v und auf der rechten Seite die Formel für das Additionstheorem mit v2=c. Dann kommt man natürlich auf v=0.

    Für meine „Widerlegung des Additionstheorems“ Einsteins braucht man keine Mathematik einzusetzen, man kann sie so ohne die mathematische Sprache in einer beliebigen natürlichen Sprache verstehen: Einstein geht nämlich davon aus, dass alle Beobachter eine konstante Lichtgeschwindigkeit c messen, völlig unabhängig von ihren jeweiligen eigenen Geschwindigkeiten. Dies bedeutet, dass Einstein davon ausgeht, dass alle Beobachter relativ zum Licht ruhen. Man braucht keine Mathematik, um das Postulat Einsteins der beobachterunabhängigen Geschwindigkeit des Lichts zu verstehen, die Mathematik beschreibt nur dieses Postulats, genau so wie jede natürliche Sprache es auch beschreiben kann.

  13. Zitat Axel Krüger: „Für mich (Laie) ist Ruhesystem ein zwiespältiger Begriff. Einfach assoziiert möchte ich meinen, dass ein R. ein System ist, das sich nicht bewegt. Also z.B. wenn ich auf der Erde stillstehe. Dann ist das (von mir aus gesehen, auf mich bezogen) ein/mein Ruhe- bzw. Bezugssystem. Obwohl ich mich ja mit der Erde (ihrer Drehung um sich selber) bewege.”
    .

    Sogar der Begriff „System“ selbst ist zwiespältig. Alle einzelne Objekte können nämlich auch theoretisch als „System“ definiert werden, auch zum Beispiel ein einzelner Mensch, denn auch ein einzelner Mensch besteht aus Elementen, die zueinander ruhen oder zueinander sich bewegen können. Wenn ich den Arm ausstrecke, bewegt sich zum Beispiel mein Arm relativ zu meinem Kopf oder zu meinen Füßen usw. Deshalb ist es aus meiner Sicht für Laien nicht hilfreich, den Begriff „System“ zum Verständnis der Relativitätstheorie zu verwenden, er ist undeutlich und verwirrend. Man sollte zum Verständnis der Relativitätstheorie bei dem Begriffen „Objekte“ und vor allem bei dem wichtigen Begriff „Beobachter“ bleiben, der viel anschaulicher ist und auch von Einstein benutzt wird. Zumal einzig Beobachter (mit ihren Instrumenten) Messungen vornehmen können.

    Ein “Inertialsystem” kann man zum Beispiel im Kopf ganz einfach definieren bzw. erkennen als ein Objekt oder ein Beobachter, der zu einem zweiten Objekt ruht, immer nur paarweise, die Relation ist immer nur paarweise zu untersuchen! Ich benutze so gut wie nie den Begriff “System”, sondern einzig die Begriffe „ruhender Beobachter“ oder „bewegter Beobachter“ und gebe natürlich an bzw. frage nach, zu was dieser Beobachter ruht oder bewegt ist – so viel Zeit muss sein, dass man den Bezug der Ruhe oder der Bewegung angibt, sonst ergibt es überhaupt gar keinen Sinn. Es ist kinderleicht bei einer bestimmten Konstellation zu erkennen was oder wer paarweise in Bezug zu was ruht oder bewegt ist, man kann gar keine Fehler machen.

    Zitat Forschungsgruppe G.O. Mueller:

    In den Darstellungen der Relativistik wimmelt es von Aussagen „ruhend“ und „bewegt“ ohne Angabe eines Bezugskörpers oder Bezugssystems.

    Nach dem Relativitätsprinzip der Speziellen Relativitätstheorie kann es grundsätzlich nur relative Bewegung und relative Ruhe geben, weshalb ausnahmslos alle Aussagen über Ruhe und Bewegung, seien sie durch Substantive oder Adjektive ausgedrückt, angeben müssen, in Bezug worauf etwas ruht oder sich bewegt.

    Der Bezug kann ausdrücklich angegeben werden oder auch, wenn der Sachzusammenhang eng und eindeutig ist, über eine längere Passage hin gelten, muß jedoch für jeden Leser zweifelsfrei zu erkennen sein; auf keinen Fall jedoch dürfen beziehungs- und bezugslos stehende Angaben über „Ruhe“ oder „Bewegung“ toleriert werden.
    .

    Über diese Forderung nach eindeutigem Bezug für jede Ruhe-/Bewegung-Aussage hat sich bereits Albert Einstein in seiner ersten Veröffentlichung 1905 häufig hinweggesetzt, wie in den Fehlern des Abschnitts E – Bewegung nachgewiesen. In fast allen Darstellungen der Relativistik findet sich ein mehr oder weniger undisziplinierter Gebrauch von Aussagen über „Ruhe“ und „Bewegung„, die jeder Leser selbst rot anstreichen kann und zum Schluß auszählen kann, wenn er will. Jede rot angestrichene Stelle stellt einen Fehler in der Darstellung und folglich in der Argumentation dar.“

  14. @ Rudi Knoth – heute – 11:57

    Ihr Bsp. des Stabes mit zwei Spiegeln und einer Lichtquelle in der Mitte wird in diesem Video https://www.youtube.com/watch?v=ev9zrt__lec als Link über die *Nature of Time and Simultaneity* thematisiert — bei 7:18 inks unten die Schaltfläche *Click for details*.

    Bei 5:28 im Video (nicht dem Link) wird gesagt, dass everything is slower by the exact same amount, including the speed of his thoughts. Auch eine “nette“ (interessante) Konsequenz der Zeitdilatation = dass man (sogar) langsamer denkt. Was wiederum den psychologischen/subjektiven (man könnte auch sagen einen gewissen “philosophischen“) Aspekt der Zeit anspricht (Stichwort individueller point of view) – als Gegensatz zur physikalischen/objektiven Zeit (-definition/-messung).

    Das Video selber ist etwa 16 Min. lang. Es sind darin aber div. Links (und Links aus diesen Links heraus), die die Gesamtdauer auf knapp 1 Std. erhöhen (“uff“). Plus am Ende nochmal 2 Links (End of the Universe und Quantum Mechanics) von zusammen nochmals ca. 1 Std. Wobei ich nicht weiß, ob da nicht auch noch Links enthalten sind (hab das jetzt nicht angeschaut – und auch nicht jeden der anderen Links bis zum Ende). Nichtsdesto: “Enjoy“ (ggfls.) …

  15. @Lopez 15:45

    Dies bedeutet, dass Einstein davon ausgeht, dass alle Beobachter relativ zum Licht ruhen.

    Nun genau das nicht. Es gibt nur die Aussage, dass in allen Inertialsystemen die Lichtgeschwindigkeit gleich ist. Also jeder Beobachter dei gleiche Lichtgeschwindigkeit misst. Da das Licht sich mit dieser Geschwindigkeit für jeden Beobachter bewegt, ruht er natürlich nicht gegenüber dem Licht.

  16. @Martin Holzherr

    Jeder elektrische Strom wird von einem Magnetfeld umgeben und dieses Magnetfeld entsteht durch Längenkontraktion der bewegten Elektronen relativ zu den unbewegten positiv geladenen Atomkernen des Metalls, was zu einem zusätzlichen elektrischen Feld führt, welches wir aufgrund seiner speziellen räumlichen Anordnung Magnetfeld nennen.

    Da gehen zwei Dinge durcheinander.
    Magnetfelder enstehen einfach durch fließende Ströme lassen sich mit dem Durchflutungsgesetz berechnen. Die Relativitätstheorie spielt hier keine Rolle.
    Was im Alltag sichtbar wird, ist die Lorentzkraft. Sie ist relativistischen Ursprungs, selbst bei Geschwindigkeiten, die sehr viel kleiner als die Lichtgeschwindigkeit sind.
    Sie kann mit Hilfe kontrahierter Raumladungsdichten und geeigneter Wahl des Bezugssystems auf eine elektrostatische Kraft zurückgeführt werden.

  17. @Axel Krüger 17:56

    Bei 5:28 im Video (nicht dem Link) wird gesagt, dass everything is slower by the exact same amount, including the speed of his thoughts. Auch eine “nette“ (interessante) Konsequenz der Zeitdilatation = dass man (sogar) langsamer denkt.

    Nun das ist ja die Zeitdilatation. Die Ereignisse des bewegten Objektes laufen alle langsamer. Aber dann gibt es ja mein Beispiel unter dem Motto “eine ruhende Uhr kann auch eine bewegte Uhr sein”.

    Wenn eine Raumsonde mit etwa 20% der Lichtgeschwindigkeit von der Erde nach Alpha-Centauri fliegt (DEEP-IN), dann ist im Inertialsystem der Erde die Zeit zwischen Start und Rückmeldung von der Passage bei Alpha-Centauri kürzer als im Inertialsystem der Raumsonde. Also hier ist die Zeitdilatation andersrum.

  18. Die Beiträge von Frau Lopez zeigen sehr schön, was passiert, wenn man wider besseren Wissens mechanische Systeme und Koordinatensysteme durcheinander wirft. Besser kann man den Inhalt meines Artikels nicht bestätigen.

    Es zeigt auch sehr schön, was passiert, wenn man die vor-relativistische Physik nicht beherrscht. Tatsächlich unterscheidet sich die Konstruktion eines Ruhesystems in der SRT nicht von der in der Newtonschen Physik: Man wähle einen Punkt des geradlinig, gleichförmig bewegten Körpers, für den man ein Ruhesystem definieren will. In diesem Punkt definiere man drei zueinander orthogonale Achsen als die drei Raumachsen. Die Koordinaten bewege man linear in der Zeit, so dass der Ursprung immer am gewählten Punkt bleibt. Fertig. In der newtonschen Physik lässt sich zu jedem geradlinig, gleichförmig bewegten Körper solch ein Ruhesystem definieren.

    Die Methode geht übrigens nicht auf Galileo sondern auf Descartes zurück.

  19. Zitat Joachim SchulzDie Beiträge von Frau Lopez zeigen sehr schön, was passiert, wenn man wider besseren Wissens mechanische Systeme und Koordinatensysteme durcheinander wirft. Besser kann man den Inhalt meines Artikels nicht bestätigen.
    Es zeigt auch sehr schön, was passiert, wenn man die vor-relativistische Physik nicht beherrscht. Tatsächlich unterscheidet sich die Konstruktion eines Ruhesystems in der SRT nicht von der in der Newtonschen Physik.“

    .

    Die Antwort von Joachim Schulz zeigt sehr schön, wie die Relativisten versuchen, einem Laien zu vernebeln, der auf eine Inkonsistenz bei der Geschwindigkeitsadditions Einsteins aufmerksam macht. Es zeigt auch sehr schön, dass die Relativisten grundsätzlich jedem Laien unterstellen, nicht einmal die klassische Physik zu beherrschen und eine eigene Erfahrung der Welt zu haben.

    Ich habe darauf aufmerksam gemacht, dass Einstein die Geschwindigkeitsaddition nach Galilei als ungültig erklärt, wonach die jeweiligen Geschwindigkeiten v von beliebigen relativ zum Licht bewegten Beobachtern sich zur Geschwindigkeit c des Lichts addieren. Bei Galilei/Newton gilt in der klassischen Physik eine variable Geschwindigkeit des Lichts c + v. Sie gilt heute noch, sie hat sich auch in der Empirie als richtig erwiesen, sie wird nämlich in der GPS-Technologie und in der Raumfahrt angewandt: GPS navigiert mit Galilei-Newton c + v.

    Einstein widerspricht jedoch Galilei/Newton und postuliert: Es gilt keine variable Geschwindigkeit des Lichts c + v, es gilt einzig eine konstante Geschwindigkeit c=const relativ zu beliebig zum Licht bewegten Beobachtern, völlig unabhängig von ihren jeweiligen Geschwindigkeiten v. Es gibt gar keine Geschwindigkeiten v von gar keinem bewegten Beobachter, die sich zu c addieren können, der Wert der Lichtgeschwindigkeit c ist strikt beobachterunabhängig und ändert sich nie, kein Jota. Die Geschwindigkeitsaddition Einsteins lautet: c + v = c. Das bedeutet, dass Einstein die Geschwindigkeiten aller Beobachter auf Null stellt: Es gilt immer nur v = 0.

    Ich habe dann als Laie gefragt: Wie kann es angehen, dass die Geschwindigkeit eines Beobachters v>0 in der Realität einfach so mathematisch Simsalabim verschwinden kann? Wie kann es angehen, dass ein Beobachter immer zu einem Lichtstrahl ruht, auch wenn es sich zu einem Lichtstrahl bewegt? 🙁

    Auf die Antwort der Relativisten müsste eigentlich jeder selbst kommen, der im Gegensatz zu mir nicht dumm wäre und etwas von Physik verstehen würde: Ein bewegter Beobachter ruht doch immer, er ruht doch zu sich selbst!! Man kann doch jederzeit ein bewegter Beobachter als absolut ruhend festlegen: Ein bewegter Beobachter ruht immer zu sich selbst. Und fertig ist die absolute Konstanz der Lichtgeschwindigkeit zu allen zu sich selbst ruhenden bewegten Beobachtern. Genial, oder?

    Dies hat mir der Relativist Markus Pössel 2008 in einem E-Mail-Austausch anläßlich eines Gedankenexperiments von mir erklärt: „Jeder dieser Beobachter sitzt im Nullpunkt seines eigenen, relativ zum Strand bewegten, Koordinatensystems.“

    Wenn ich mich bewege, bewege ich mich in Wirklichkeit gar nicht, ich sitze nur im Nullpunkt meines eigenen bewegten Koordinatensystems, wie in einer kleinen Kutsche. Genial, finde ich, das ist auch nicht anstrengend, mein eigenes bewegtes Koordinatensystem fährt mich überall rumspazieren, ich brauche nur bequem darin zu sitzen. Eigentlich eine tolle Erfindung, dieses Koordinatensystem, oder? 🙂

  20. @Lopez

    Ich habe dann als Laie gefragt: Wie kann es angehen, dass die Geschwindigkeit eines Beobachters v>0 in der Realität einfach so mathematisch Simsalabim verschwinden kann? Wie kann es angehen, dass ein Beobachter immer zu einem Lichtstrahl ruht, auch wenn es sich zu einem Lichtstrahl bewegt?

    Die Frage wurde Ihnen oft genug beantwortet. Um Markweger, durchaus ein Verbündeter von Ihnen, zu zitieren: “Das Schöne an der Relativitätstheorie ist ja dass sie seit 100 Jahren mehr oder weniger kontinuierlich erklärt wird.”

    Nun setzten Sie sich hin und vollziehen Sie die Erklärung nach. Das kann auch Laien niemand abnehmen.

  21. @Jocelyne Lopez 28. Juni 2018 @ 07:47

    Sie gilt heute noch, sie hat sich auch in der Empirie als richtig erwiesen, sie wird nämlich in der GPS-Technologie und in der Raumfahrt angewandt: GPS navigiert mit Galilei-Newton c + v.

    Das ist Unsinn. Bei Doppelsternen weis man seit über 80 Jahren, daß das Licht der Komponenten immer mit c hier ankommt. Und was GPS angeht, woher haben Sie dies? Zwar wird von t=t’ gesprochen, aber immer angemerkt, daß die Uhren der Satelliten so eingestellt sind, daß sie mit Uhren im ECI synchron sind. Oder meinen Sie den Sagnac-Effekt?

    Die Geschwindigkeitsaddition Einsteins lautet: c + v = c. Das bedeutet, dass Einstein die Geschwindigkeiten aller Beobachter auf Null stellt: Es gilt immer nur v = 0.

    Nein das stimmt nicht. Sie vermischen hier Galilei und Lorentz/Einstein.

  22. Zitat Joachim Schulz: “Die Frage wurde Ihnen oft genug beantwortet. Um Markweger, durchaus ein Verbündeter von Ihnen, zu zitieren: “Das Schöne an der Relativitätstheorie ist ja dass sie seit 100 Jahren mehr oder weniger kontinuierlich erklärt wird. Nun setzten Sie sich hin und vollziehen Sie die Erklärung nach. Das kann auch Laien niemand abnehmen.”
    .

    Der Teilnehmer Markweger ist kein “Verbündeter” von mir, er ist ein kritischer Diskutant, der in Ihrem Blog gerade nicht gesperrt wurde, wie zum Beispiel die kritischen Diskutanten und Profi Physiker und Mathematiker Dr. Wolfgang Engelhardt und Cryptic gesperrt wurden.

    Und alle kritischen Diskutanten können seit 10 Jahren genauso wenig wie ich die “Erklärung nachvollziehen“, dass ein zum Licht bewegter Beobachter in Wirklichkeit zum Licht ruht, weil er “im Nullpunkt seines eigenen bewegten Koordinatensystems sitzt“. 🙁

  23. @Lopez heute 10:19

    Und alle kritischen Diskutanten können seit 10 Jahren genauso wenig wie ich die “Erklärung nachvollziehen“, dass ein zum Licht bewegter Beobachter in Wirklichkeit zum Licht ruht, weil er “im Nullpunkt seines eigenen bewegten Koordinatensystems sitzt“.

    Der Beobachter ruht sicher nicht zum Licht, denn das bewegt sich relativ zu ihm mit c. Und mit Ihrer Ausssage über das “ruhen im Koordinatensystem” stellen Sie die Fakten auf de Kopf. Das Koordinatensystem bewegt sich mit dem Beobachter, weshalb er natürlich sich gegenüber diesem nicht bewegt. Dies gilt aber, wie Herr Schulz sagt, auch bei Galilei etc.

  24. Dass sich “Koordinatensysteme” wie physikalische Objekte in der Natur bewegen können, hat Albert Einstein himself in seinem ersten Postulat festgelegt:

    Zitat Albert Einstein: „1. Die Gesetze, nach denen sich die Zustände der physikalischen Systeme ändern, sind unabhängig davon, auf welches von zwei relativ zueinander in gleichförmiger Translationsbewegung befindlichen Koordinatensystemen diese Zustandsänderungen bezogen werden. [Hervorhebung durch Lopez].

    Wir haben schon in der SRT die Verdinglichung des mathematischen Konstrukts „Raumzeit“ von Hermann Minkowsky zu schlucken, die sich krümmen, bewegen, drehen, stauchen, dehnen, wellen, zittern, vibrieren oder pulsieren kann, oder auch kippen kann, oder auch mit Löchern durchbohrt werden kann. Wir haben direkt von Albert Einstein die Verdinglichung des mathematischen Konstrukts „Koordinatensystem“.

    Jetzt sollten hier eigentlich die Philosophen, Erkenntnistheoretiker und ggfs. die Psychologen dem rationellen Normalbürger erklären, wie ein mathematisches Konstrukt wie ein “Koordinatensystem” sich als physikalisches Objekt verdinglichen kann und in die Landschaft spazieren gehen darf, um ein anderes spazierendes Koordinatensystem zu begegnen: Magie? Wunder? Esoterik? Scherz? Mathematische Spielerei? Spinnerei? Verarschung?

  25. @Lopez 28. Juni 2018 @ 11:53

    Wir haben schon in der SRT die Verdinglichung des mathematischen Konstrukts „Raumzeit“ von Hermann Minkowsky zu schlucken, die sich krümmen, bewegen, drehen, stauchen, dehnen, wellen, zittern, vibrieren oder pulsieren kann, oder auch kippen kann, oder auch mit Löchern durchbohrt werden kann. Wir haben direkt von Albert Einstein die Verdinglichung des mathematischen Konstrukts „Koordinatensystem“.

    Diese Koordinatensysteme bewegen sich aber mit materiellen Objekten. Ein von reellen Objekten sich bewegendes Koordinatensystem ist ja auch nicht sinnvoll, denn man will ja die räumlichen/zeitlichen Grössen zwischen zwei Objekten wissen.

    Jetzt sollten hier eigentlich die Philosophen, Erkenntnistheoretiker und ggfs. die Psychologen dem rationellen Normalbürger erklären, wie ein mathematisches Konstrukt wie ein “Koordinatensystem” sich als physikalisches Objekt verdinglichen kann und in die Landschaft spazieren gehen darf, um ein anderes spazierendes Koordinatensystem zu begegnen: Magie? Wunder? Esoterik? Scherz? Mathematische Spielerei? Spinnerei? Verarschung?

    Jetzt werden Sie aber lustig. Wie ich oben sagte, sind ja diese Koordinatensysteme mit Objekten verbunden. Auf der Erde gibt es ja auch ein Polarkoordinatensystem, dass man Länge, Breite und Höhe nennt. Es dreht sich natürlich mit der Erde.

  26. Die Art, wie Einstein Koordinaten benutzt, ist keine Erfindung von Ihm. Es war bereits eine etablierte Methode der theoretischen Mechanik. In dem von Ihnen, Frau Lopez, zitierten Text, ruft Einstein einfach das bereits bekannte Postulat von Galileo ins Gedächtnis. Dass Naturgesetze vom Bewegungszustand unabhängig sind, war damals Stand der Wissenschaft. Darauf baut Einstein seine Veröffentlichung auf.

    Dass Sie keine Ahnung von klassischer Physik haben, ist keine Unterstellung von mir. Es ist einfach evident aus jedem einzelnen Beitrag, den Sie hier schreiben.

  27. Zitat Rudi Knoth: “Diese Koordinatensysteme bewegen sich aber mit materiellen Objekten. Ein von reellen Objekten sich bewegendes Koordinatensystem ist ja auch nicht sinnvoll, denn man will ja die räumlichen/zeitlichen Grössen zwischen zwei Objekten wissen.”
    .
    Gut, Herr Knoth, Sie sind studierter Physiker, also gehören Sie zu den wenigen Auserwählten in dieser Welt, die in der Lage sind, fachlich und intellektuell die Relativitätstheorie zu verstehen, was man von mir leider nicht behaupten darf… 🙁

    Dann schlage ich ein Gedankenexperiment mit einer einfachen Konstellation vor:

    Ich laufe auf meiner Straße zum Bäcker mit Geschwindigkeit v=4 km/h, um Brötchen zu kaufen.

    1. Gemäß SRT laufe ich nicht zum Bäcker, sondern ich sitze ruhend und kräftefrei im Nullpunkt meines sich zum Bäcker bewegenden eigenen Koordinatensystems. Ich habe also nicht die Geschwindigkeit v = 4 km/h, sondern die erholsame Geschwindigkeit v = 0 km/h.

    2. Mein eigenes Koordinatensystem, das mich zum Bäcker mitführt, ruht jedoch nicht: Es bewegt sich mit v = 4 km/h Richtung Bäcker. Es bewegt sich auch nicht kräftefrei wie ich, denn – wie der Teilnehmer Axel Krüger es weiter oben richtig angemerkt hat – bewegt sich nichts ohne zumindest am Anfang eine Kraft erfahren zu haben. Also irgendetwas hat mein sich bewegendes eigenes Koordinatensystem geschubst, oder es verfügt irgendwie vielleicht über einen kleinen Motor, weiß man nicht, lassen wir offen.

    Ich schließe jedoch daraus, dass ich mich doch selbst als Mensch irgendwie in Richtung Bäcker bewege und dass ich irgendwann dort selbst meine Brötchen kaufen werde – ich kann es doch nicht von meinem eigenen Koordinatensystem verlangen, zumal es mit Brötchen nichts anfangen kann.

    Erweitern wir das Gedankenexperiment:

    Mein Nachbar aus der anderen Ende der Straße möchte auch Brötchen beim Bäcker kaufen und joggt dahin mit 10 km/h. Er joggt eigentlich gar nicht, natürlich, er sitzt auch bewegungslos und kräftefrei im Nullpunkt seines eigenes, sich zum Bäcker bewegenden Koordinatensystem.

    Unsere beiden jeweils eigenen Koordinatensysteme bewegen sich also auf der Straße zueinander, das meine mit 4 km/h, das von meinem Nachbar mit 10 km/h.

    Die Vorstellung Einsteins in diesem Fall besteht darin, dass dasjenige Bezugsobjekt, auf welches man die Geschwindigkeit des anderen bezieht, automatisch die Geschwindigkeit Null erhält.

    Was kann man hier logisch ableiten?

    1. Wenn ich und mein Nachbar jeweils im Nullpunkt unseren eigenen Koordinatensystemen ruhen, kann man ableiten, dass wir zueinander ruhen. Keiner von uns beiden bewegt sich, wir bleiben stehen, wir werden nie den Bäcker erreichen.

    2. Man kann auch ableiten, dass ich allein mich mit 4 km/h zum Bäcker bewege und dass mein Nachbar stehen bleibt, Pech. Ich allein werde also Brötchen kaufen können.

    2. Man kann auch ableiten, dass allein mein Nachbar sich zum Bäcker mit 10 km/h bewegt und er allein Brötchen kaufen wird, ich bleibe einfach da stehen, diesmal Pech für mich.
    .

    Wie man sieht, das ist gar nicht so einfach mit der Relativitätstheorie Brötchen zu kaufen, wenn überhaupt möglich. 🙁 

    Zum Glück habe ich die klassische Physik mit Galilei/Newton, no problem um Brötchen zu kaufen: Ich laufe selbst mit Muskelkraft mit 4 km/h zum Bäcker und kaufe meine Brötchen, mein Nachbar läuft selbst mit Muskelkraft mit 10 km/h zum Bäcker und kauft seine Brötchen, wir bewegen uns zueinander auf der Straße mit einer Relativgeschwindigkeit von 4 km/h + 10 km/h = 14 km/h. Ist die klassische Physik nicht herrlich für Menschen, die weder fachlich noch intellektuell die Relativitätstheorie verstehen können? 🙂

  28. @Jocelyne Lopez 28. Juni 2018 @ 15:44

    Das Koordinatensystem sei mit Ihnen. Weder Sie noch Ihr Nachbar ruhen relativ zum Bäcker, sondern bewegen sich jeweils mit 4 Km/h und 10 Km/h in Richtung des Bäckers. Da Ihr Koordinatensystem dieselbe Richtung und Geschwindigkeit wie Sie hat, benutzt man den Begriff “ruhend”. Dasselbe gilt für Ihren Nachbarn. Auch in seinem Koordinatensystem ruht er. Aber Sie wie Ihr Nachbar bewegen sich im Koordinatensystem des Bäckers. Wie Herr Schulz schrieb, gab es diese sich bewegenden Koordinatensystem schon bei Newton/Galilei.

  29. @oachim Schulz 28. Juni 2018 @ 17:08

    Ist das Satire?
    Bei mir kommt übrigens eine Relativgeschwindigkeit von 6km/h heraus. Egal ob ich relativistisch oder galileisch rechne.

    Nun ich habe das so verstanden:

    Mein Nachbar aus der anderen Ende der Straße möchte auch Brötchen beim Bäcker kaufen und joggt dahin mit 10 km/h.

    Also ihr Nachbar joggt vom anderen Ende der Strasse zum Bäcker. Also beide Personen laufen zueinander zu. Dann kommen 14 Km/h als Differenz- oder Relativgeschwindigkeit zustande.

  30. Oh, Sie waren schneller als ich meinen Beitrag löschen konnte. Also gleich wieder aus dem Papierkorb rausgeholt.

    Ich bin jetzt wirklich am grübeln, ob uns Frau Lopez nur zum Narren hält. So schwierig kann es doch nicht sein, einzusehen, dass diese Relativität schon in der Newtonschen Mechanik enthalten ist.

  31. Zitat Joachim Schulz: “Die Art, wie Einstein Koordinaten benutzt, ist keine Erfindung von Ihm. Es war bereits eine etablierte Methode der theoretischen Mechanik. In dem von Ihnen, Frau Lopez, zitierten Text, ruft Einstein einfach das bereits bekannte Postulat von Galileo ins Gedächtnis. Dass Naturgesetze vom Bewegungszustand unabhängig sind, war damals Stand der Wissenschaft. Darauf baut Einstein seine Veröffentlichung auf. […] So schwierig kann es doch nicht sein, einzusehen, dass diese Relativität schon in der Newtonschen Mechanik enthalten ist.”

    .
    Aber Herr Schulz, ich kann das einsehen: Sie sollten vielleicht meine Beiträge gewissenhafter und aufmerksamer lesen, ich habe es doch selbst weiter oben gesagt, sogar in zwei verschiedenen Beiträgen, dass Einstein das Relativitätsprinzip Galileis für seine SRT übernommen hat, ich habe sogar darüber wortwörtlich sein 1. Postulat zitiert.

    Nur, dass die Geschwindigkeitsaddition Galileis ganz anders ist, als die Geschwindigkeitsaddition Einsteins:

    – Bei der Geschwindigkeitsaddition Galileis addieren sich die jeweiligen Geschwindigkeiten v von zwei zueinander bewegten Beobachtern zur Berechnung der Relativgeschwindigkeit zwischen den beiden: Es gilt c +/- v.

    – Bei der Geschwindigkeitsaddition Einsteins addiert sich nicht die Geschwindigkeit v eines Beobachters zu der Geschwindigkeit v eines anderen Beobachters, jeder Beobachter erhält automatisch wahlweise und abwechselnd bei der Addition Einsteins die Geschwindigkeit v=0. Es gilt c + v = c.

    Ich habe gar kein Problem mit der Geschwindigkeitsaddition Galileis – und überhaupt mit der klassischen Physik. Dafür ein riesiges Problem mit der Geschwindigkeitsaddition Einsteins.

  32. @Jocelyne Lopez 28. Juni 2018 @ 18:12

    – Bei der Geschwindigkeitsaddition Einsteins addiert sich nicht die Geschwindigkeit v eines Beobachters zu der Geschwindigkeit v eines anderen Beobachters, jeder Beobachter erhält automatisch wahlweise und abwechselnd bei der Addition Einsteins die Geschwindigkeit v=0. Es gilt c + v = c.

    Das ist Unsinn. Es stimmt, daß es keine einfache Addition wie bei Galilei gibt. Wenn beide Geschwindigkeiten klein sind, ist der Unterschied zu Galilei gering. Im Fall, daß eine der Geschwindigkeiten c ist, kommt als Ergebnis c raus egal wie groß die andere Geschwindigkeit ist.Dies liegt an der Relativität der Gleichzeitigkeit.

  33. Bei Einstein ist c+v=c+v, der hat in der Schule aufgepaßt und macht keine Fehler.
    c+v ist im rotierenden System einfach Sagnac als Laufzeiteffekt, eben c=const.
    Die relativistischen Addition nach Einstein lautet ordentlich notiert c ⊕v=c.
    Diese Additionsoperation beinhaltet den Gamma-Faktor, das Schul”+” nicht.
    Irgendwo scheint Frau Lopez irgendwas verpaßt zu haben, mindestens in Mathematik.
    Das gilt für jeden Beobachter unabhängig von v. Haben wir 2 Beobachter, die sich
    relativ mit v bewegen und einer beobachtet ein Objekt mit der Geschwindigkeit u,
    dann mißt der andere w=v⊕u<c und eben nicht v+u bei großen Geschwindigkeiten.
    Hab allein ich in verschiedenen Varianten vorexerziert, wo Frau Lopez ihre obige
    Privatmathematik verkaufen wollte, ich weiß jetzt, was Begriffstutzigkeit so macht.
    Das eine heißt Addition (Schule für Zahlen und sowas), das andere Additionstheorem,
    gilt für Geschwindigkeiten nahe c=LG (Natur) und ist experimentell bestätigt.

  34. Ich fürchte, die Frau L. wird sich den Strohmann, den sie anstelle der wirklichen Aussagen der SRT so schön verbrennen kann, nicht so leicht wegnehmen lassen. Versetzen Sie sich mal in ihre Lage. Da gibt es eine Theorie, die sie so gerne widerlegen möchte. Leider ist diese Theorie mathematisch konsistent und experimentell gut belegt. Aber da kommt jemand und behauptet, die Theorie sage unter anderem c=c+v aus. Ein gefundenes Fressen. Eidlich eine Aussage, die sich leicht widerlegen lässt. Das sie nicht wirklich Teil der Theorie ist spielt keine Rolle. So wir es ja auch keine Rolle spielt, dass all die vielen Argumente der G.O. Möller Brüder längst widerlegt sind. Sie sind es halt noch nicht von jedem.

  35. Zitat Rudi Knoth: „Das ist Unsinn. Es stimmt, daß es keine einfache Addition wie bei Galilei gibt. Wenn beide Geschwindigkeiten klein sind, ist der Unterschied zu Galilei gering. Im Fall, daß eine der Geschwindigkeiten c ist, kommt als Ergebnis c raus egal wie groß die andere Geschwindigkeit ist. Dies liegt an der Relativität der Gleichzeitigkeit.“
    .
    Dies liegt nicht an der Relativität der Gleichzeitigkeit, dies liegt daran, dass Einstein eben postuliert hat (2. Postulat), dass alle Beobachter die gleiche Geschwindigkeit c messen, egal ob sie relativ zum Lichtstrahl ruhen oder ob sie sich relativ zum Lichtstrahl bewegen, egal was für eine Geschwindigkeit v sie haben, auch eine geringe, das spielt gar keine Rolle: Die beliebigen eigenen Geschwindigkeiten aller Beobachter im Universum addieren sich nicht zur Geschwindigkeit des Lichtes, nie, kein Jota, sie werden alle auf 0 gesetzt. Die hochtrabende Geschwindigkeitsaddition Einsteins enthält zwar ausdrücklich den Term c + v, jedoch ist v dabei immer als v = 0 vorausgesetzt, keine andere Größe! Die Geschwindigkeitsaddition Einsteins ist und bleibt eine Addition mit 0. Da ist Raum weder für kleine v noch für große v. Null ist Null in der Mathematik, und keine andere Zahl, genauso wie 1 ist 1 und keine andere Zahl, genauso wie 2 ist 2 und keine andere Zahl, genauso wie 299.792.458 ist 299.792.458 und keine andere Zahl.

  36. Nun da habe ich wohl was angerichtet. Zuerst einmal meine Sicht auf das Additionstheorem der Geschwindigkeiten. Man kann dies dadurch gewinnen, wenn man etwa die totale Ableitung im Messystem dx/dt bestimmt und dabei die Differentiale von dt’ abhängig macht in der Form dx/dt = (dx/dt’)/dt/dt’). Daraus erhält man dann das Additionstheorem. Man kann es auch anders herleiten. Wichtig ist dabei folgendes:

    1. Der Faktor Gamma hat in dieser Formel nichts zu suchen, weil dieser in beiden Differentialen vorkommt.

    2. Wenn man sich den Term im Nenner v1*v2/c**2 ansieht, so ist v1 die Ableitung von x’ nach t’. Es ist also der Term, der die Relativität der Gleichzeitigkeit ausdrückt (x’*v/c**2).

    Nun meine Kommentare zu Herrn Senf und Frau Lopez.

    @Herr Senf 28. Juni 2018 @ 21:04

    Nun das ist eine gute Antwort auf Frau Lopez und das mit Gamma sehe ich anders. Das sollte aber keine Feindschaft begründen. Aber mit der Adition der Geschwindigkeiten hadert sie immer noch.

    @Frau Lopez gestern Abend und heute Früh

    Dies liegt nicht an der Relativität der Gleichzeitigkeit, dies liegt daran, dass Einstein eben postuliert hat (2. Postulat), dass alle Beobachter die gleiche Geschwindigkeit c messen, egal ob sie relativ zum Lichtstrahl ruhen oder ob sie sich relativ zum Lichtstrahl bewegen,…

    Nun dann etwas Aufklärung über die SRT. Es stimmt, daß das Postulat 2 Ausgangspunkt seiner Theorie ist. Daraus folgt zwingend die “Relativität der Gleichzeitigkeit”. Wegen c=const und der endlichen Geschwindigkeit des Lichtes sieht ein Beobachter der zwischen zwei Punkten, an denen zwei gleichzeitige Ereignisse stattfinden, diese als gleichzeitig. Ein relativ zu dieser Strecke bewegter Beobachter, der zum selben Zeitpunkt “neben” dme in der Mitte sich befand, sieht diese Ereignisse nicht als gleichzeitig. Dieses wird in dem Term xv/c**2 ausgedrückt.

    egal was für eine Geschwindigkeit v sie haben, auch eine geringe, das spielt gar keine Rolle: Die beliebigen eigenen Geschwindigkeiten aller Beobachter im Universum addieren sich nicht zur Geschwindigkeit des Lichtes, nie, kein Jota, sie werden alle auf 0 gesetzt.

    Nun sie werden nicht auf Null gesetzt. Nur die Summe der Geschwindigkeiten erreicht nicht die Lichtgeschwindigkeit.

    Einstein hat es so ausgelegt, dass bei zwei zueinander bewegten Objekten ein der Objekte zwangläufig ruht während das andere sich bewegt. Er hat nicht die unzähligen Fälle dabei berücksichtigt, wo beide mit jeweils eigener Geschwindigkeiten v sich zueinander bewegen!

    Zu welchem Bezugssystem denn? Und das man eines der Systeme als ruhend annimmt, ist wohl aus dem Relativitätsprinzip logisch.

    Einstein hat bei seinem 2. Postulat die Hypothese von Lorentz/Poincaré falsch ausgelegt, wonach die Lichtgeschwindigkeit relativ zum ÄTHER konstant sei. Er hat diese postulierte Konstanz der Lichtgeschwindigkeit relativ zum ÄTHER in eine postulierte Konstanz der Lichtgeschwindigkeit relativ zum BEOBACHTER kurzerhand verwandelt! Etwas ganz Anderes!

    Nun er hat einfach eine andere Schlussfolgerung aus den experimentellen Befunden (Michelson-Morley) gezogen. Um diese Befunde zu erklären, hatte Lorentz dem Äther die Eigenschaft zugeschrieben, durch Veränderungen der Instrumente sich des Nachweises zu entziehen. Es ist also nach Lorentz nicht möglich, Bewegungen relativ des Äthers zu bestimmen. Er hat also eine “magische” Annahme fallen gelassen.

  37. Hi Jocelyne,

    die spezielle RT wirft weniger mathematische oder physikalische Fragen auf als vielmehr metaphysische.

    Da ist erstmal die Frage, was kann oder soll sich überhaupt mit Lichtgeschwindigkeit bewegen ?

    Lt Physik ist dies (Licht)Strahlung, die man als elm. Welle oder Schauer von Lichtquanten, sog. Bosonen, bezeichnen kann, aber das sind reine Modellvorstellungen. Mit der ontischen Realität haben diese Modelle nicht viel zu tun, denn elm. Wellen sind nur mathematische Konstrukte, und Licht ist ein physikalisches Phönomen und kann daher nicht aus mathematischen Konstrukten bestehen; und Bosonen sind nun mal masselose Teilchen mit definiertem Impuls, diese darf es als Punktteilchen wegen der Heisenbergschen Unschärferelation in der Realität ebenso wenig geben. Also was bewegt sich mit Lichtgeschwindigkeit c ? Wie sollen wir wissen, daß sich etwas mit Lichtgeschwindigkeit bewegt, wenn wir nicht wissen, was dieses Etwas ist ? Vor allem wissen wir nicht, welchen Weg dieses Etwas geht. Sollte seine „Bewegung“ mit c vom Ort seiner Entstehung bis zum Ort seiner Wirkung gar nicht auf dem kürzestem Weg verlaufen, sondern auf einem Kreisbogen, würden wir immer noch c messen, aber in Wirklichkeit wäre die Geschwindigkeit dieses Etwas viel größer als c. Und um bei Thema zu bleiben: hat ein lichtgeschwindigkeitsbewegtes „System“ (falls es so was gibt) ein Bezugssystem, oder kann man eines (meinetwegen im Grenzfall) definieren ? Meiner Ansicht nach nicht – und wenn nicht, kann man dann ein „System“ ohne Bezugssystem überhaupt aus einem Bezugssystem heraus beschreiben ?

    Da steht einiges auf sehr wackeligem Beschreibungsuntergrund und man kann da viel spekulieren, was der Güte der Theorie aber keinen Abbruch tut. Vor allem aber stellt sich doch die Frage: warum gibt es überhaupt eine Maximalgeschwindigkeit (wenn denn c als Wert mit Einheit überhaupt eine Geschwindigkeit ist). Es gibt keine physikalische Theorie, aus der sie sich ableiten ließe, auf die warten wir noch. C ist insofern ein echtes unverstandenes physikalisches Phänomen. Man kann sich einem Verständnis von c aber von zwei Seiten nähern:

    a) C als Spezialfall einer allgemeinen beschränkten universellen Veränderungsrate. Immerhin hat das Universum eine Ordnung und diese würde zusammenbrechen (behaupte ich), wenn es interne Unendlichkeiten gäbe. Also muß die Rate, mit der prozessurale Veränderungen von statten gehen, generell beschränkt sein. Es wäre von Interesse zu erfahren, ob es dafür (außer c) noch andere Anhaltspunkte gibt in der Physik, z.B. die endliche Dauer von Energiesprüngen angeregter Atome. C wäre dann nur die maximale Ortswechselrate, also ein auf Ortswechsel reduzierte Spezialfall.

    b) Interessanter ist eine andere Überlegung, wo c herkommen könnte: Über Strahlung wissen wir ontologisch nichts, wir wissen aber: es entsteht etwas an einem Punkt – und danach tritt eine Wirkung an einem anderen Punkt auf. Also ein Vorher, Nachher und zwei Punkte. Genau dadurch entstehen (im Sinne einer Genese) Abstände: Zeit- und Raumabstände. Erst entstehen die beiden Punkte (Punkte, wo etwas entsteht und vergeht), und erst danach habe ich auch die Abstände. Das sieht so aus, als ob hier Raum und Zeit quasi entsteht, also durch den Prozeß des Entstehens und Vergehens von Strahlung quasi aufgespannt wird. Dieses Aufspannen von Raum und Zeit muß über einen Parameter gesteuert werden, das macht c. C strukturiert diesen Entstehungsprozess. Die Raum und Zeitabstände selbst sind nicht primär (Einsteins genialer Geistesblitz), sondern Folge eines Prozesses, den wir Strahlung nennen, und insofern auch nicht absolut zu sehen, sondern einfach nur Folge einer vorgehenden Dynamik, die von c beherrscht wird.

    Ist alles sehr spekulativ, aber die Physik hat auch nicht anderes, jedenfalls keine Ursache dafür, dass es eine Maximalgeschwindigkeit gibt, und gerade diese.
    Also Sie können herrlich spekulieren, aber nicht physikalisch mathematisch gegen Einstein, das bringt uns einfach nicht weiter, da braten wir nur im eigenen Saft. Die Theorie gibt doch super Denkanstöße.

    Grüße Fossilium

  38. @fossilium 29. Juni 2018 @ 09:56

    die spezielle RT wirft weniger mathematische oder physikalische Fragen auf als vielmehr metaphysische.

    Nun das ist erstmal hier ein Blog über Physik. Sicher gibt es mit der RT und wie ich aus Ihrem Text entnehme Fagen über die Natur der Dinge “an sich”. Nun meine bescheidenen Erklärungsversuche:

    Lt Physik ist dies (Licht)Strahlung, die man als elm. Welle oder Schauer von Lichtquanten, sog. Bosonen, bezeichnen kann, aber das sind reine Modellvorstellungen.

    Nun der Welle-Teichen Dualismus der Quantentheorie trifft auch auf Elektronen un Neutronen zu.

    Mit der ontischen Realität haben diese Modelle nicht viel zu tun, denn elm. Wellen sind nur mathematische Konstrukte, und Licht ist ein physikalisches Phönomen und kann daher nicht aus mathematischen Konstrukten bestehen;

    Nun wenn Sie mit ihrem Notebook Ihren Beitrag geschrieben haben, dann haben Sie mit dem WLAN auch ein mathematisches Konstrukt zum Ansenden benutzt. Die Welleneigenschaft des Lichtes ist aber schon lange bekannt. Das sehen Sie wenn sich sich eine DVD oder CD anschauen.

    und Bosonen sind nun mal masselose Teilchen mit definiertem Impuls, diese darf es als Punktteilchen wegen der Heisenbergschen Unschärferelation in der Realität ebenso wenig geben.

    Nun masselos sind diese Teilchen eher im Sinne von Ruhemasse Null. Und wer spricht denn von Punktteilchen?

    Sollte seine „Bewegung“ mit c vom Ort seiner Entstehung bis zum Ort seiner Wirkung gar nicht auf dem kürzestem Weg verlaufen, sondern auf einem Kreisbogen, würden wir immer noch c messen, aber in Wirklichkeit wäre die Geschwindigkeit dieses Etwas viel größer als c.

    Nun das Fermatsche Prinzip (Bewegung immer auf dem Zeitlich kürzesten Weg) schliesst dies aus. In einem einheitlichen Medium bewegt sich dieses Etwas geradlinig.

    Und um bei Thema zu bleiben: hat ein lichtgeschwindigkeitsbewegtes „System“ (falls es so was gibt) ein Bezugssystem, oder kann man eines (meinetwegen im Grenzfall) definieren ? Meiner Ansicht nach nicht – und wenn nicht, kann man dann ein „System“ ohne Bezugssystem überhaupt aus einem Bezugssystem heraus beschreiben ?

    Gute Frage, nur welches Objekt soll denn dieses System haben?

    Ist alles sehr spekulativ, aber die Physik hat auch nicht anderes, jedenfalls keine Ursache dafür, dass es eine Maximalgeschwindigkeit gibt, und gerade diese.

    Nun die Physik kann Experimente durchführen und Messungen vornehmen. Wenn es Ergebnisse aus diesen Experimenten gibt, die dieser Maximalgeschwindigkeit für Materie, Energie und Information widersprechen, können wir nochmal diskutieren. Ich denke dann diskutiert auch Herr Shulz mit anderen Physikern. Wichtig ist, daß man von Maximalgeschwindigkeit für Materie, Energie oder Information spricht Denn sonst kommt kemand mit einem Laserpointer an, streicht schnell einmal über den Mond uns sagt dann “Der Fleck hat Überlichtgeschwindigkeit”.

  39. Hallo Herr Knoth,

    ich denke, man treibt Physik, wenn man (aus dem mathematischen Formalismus einer Theorie) Modellvorstellungen ableitet, z.B. über einen Mechanismus zur Herbeiführung eines Meßwertes oder eines Ereignisses – und man treibt Metaphysik, wenn man sagt, woraus die Realität besteht, also das beschreibt, was tatsächlich in der Wirklichkeit gegeben ist. Dieser Unterschied ist nun mal sehr wichtig !

    Im metaphysischen Diskurs muß man z.B. strenge Regeln einhalten, man darf nicht widersprüchlich argumentieren. Diese strengen Regeln gelten in der Physik, z.B. bei der Beschreibung von Modellen, nicht. Da Sie diese strikte Trennung nicht vornehmen, gehen Ihre o.a. Einwände gegen meine Behauptungen entweder an der Sache vorbei, oder sind selbst voller Widersprüche. Ich will darauf aber nicht eingehen.

    Denn Sie haben Recht, man sollte beim Thema bleiben. Und hier würde mich eben sehr interessieren, was die Physik zu einem Koordinatensystem sagt, das sich mit Lichtgeschwindigkeit c an einem ruhenden Koordinatensystem vorbeibewegt, z.B. einem, in dem wir uns als Beobachter befinden, falls Sie sich das vorstellen können. Nun aus irgendwas wird Licht ja bestehen, es hat ja eine Wirkung, wenn ich die Hand in die Sonne halte, wird sie vom Licht warm. Also ist da was – was auch immer – und kann ich diesem Etwas ein Bezugssystem zuordnen ? Wenigstens in einem Grenzübergang ?

    Falls ja, hätten die Objekte, z.B. Strahlungsquanten, die sich mit dem System mitbewegen, die Eigenzeit Null, und aus aus diesem Bezugssystem betrachtet schrumpft das Universum auf einen Punkt zusammen. Die Hintergrundstrahlung, die aus unserem Ruhesystem betrachtet etliche Mrd. Jahre alt ist, würde im eigenen mit c bewegten Bezugssystem im gleichem Moment am gleichen Ort entstehen und gleichzeitig schon wieder vergehen, das Universum, das sie durcheilt, besteht aus einem auf Null geschrumpften Massepunkt. Insofern ist es eigentlich nicht falsch, davon zu sprechen, daß so ein Bezugssystem singulär ist. Allerdings hat diese Singularität einen meßbaren scharfen Impuls, und aus dem Ruhsystem betrachtet müßte das Licht dann maximal ortsunscharf sein, hat aber eine scharfe, lokale Wirkung.

    Ich finde das nicht schlimm, richtige Vorhersagen sind von diesen Problemen nicht tangiert. Aber irgendwie paßt das an dieser Stelle nicht – die Theorie muß hier an irgendeine Grenze stoßen, hat aber an dieser Grenze, nämlich mit c bewegten Objekten, mathematisch und modellhaft keine Probleme. Ich meine, es muß erlaubt sein, hier Fragen zu stellen. Natürlich kann ich mich irren – so fit bin ich in der SRT auch nicht. Immerhin darf man das merkwürdig finden.

    Grüße Fossilium

  40. @fossilium 29. Juni 2018 @ 14:18

    ich denke, man treibt Physik, wenn man (aus dem mathematischen Formalismus einer Theorie) Modellvorstellungen ableitet, z.B. über einen Mechanismus zur Herbeiführung eines Meßwertes oder eines Ereignisses – und man treibt Metaphysik, wenn man sagt, woraus die Realität besteht, also das beschreibt, was tatsächlich in der Wirklichkeit gegeben ist. Dieser Unterschied ist nun mal sehr wichtig !

    Und was bedeuten denn die Begriffe und “tatsächlich” und “Wirklichkeit”? Man kann von einer “verborgenen” Wirklichkeit sprechen oder von einer Wirklichkeit, die durch Wirkungen also Messungen oder Beobachtungen ermittelt wird. Nehmen wir die Längenkontraktion. Der Schatten eines bewegten Objektes ist kürzer als das Objekt selbst. Also ist das Objekt wirklich kürzer, obwohl es in “seinem” Koordinatensystem seine vorgegebene Länge hat. Welche Länge ist dann wirklich?

    Diese strengen Regeln gelten in der Physik, z.B. bei der Beschreibung von Modellen, nicht.

    Beispiele bitte? Aus den Modellen kann man Ergebnisse für Experimente und Messwerte ableiten.

    Nun aus irgendwas wird Licht ja bestehen, es hat ja eine Wirkung, wenn ich die Hand in die Sonne halte, wird sie vom Licht warm. Also ist da was – was auch immer – und kann ich diesem Etwas ein Bezugssystem zuordnen ? Wenigstens in einem Grenzübergang ?

    Nun die Natur des Lichtes kann beschrieben werden. Man hat die Maxwellgleichungen und die Quantentheorie, mit denen man Vorrausagen für Experimente machen kann.

    Falls ja, hätten die Objekte, z.B. Strahlungsquanten, die sich mit dem System mitbewegen, die Eigenzeit Null, und aus aus diesem Bezugssystem betrachtet schrumpft das Universum auf einen Punkt zusammen. Die Hintergrundstrahlung, die aus unserem Ruhesystem betrachtet etliche Mrd. Jahre alt ist, würde im eigenen mit c bewegten Bezugssystem im gleichem Moment am gleichen Ort entstehen und gleichzeitig schon wieder vergehen, das Universum, das sie durcheilt, besteht aus einem auf Null geschrumpften Massepunkt. Insofern ist es eigentlich nicht falsch, davon zu sprechen, daß so ein Bezugssystem singulär ist.

    Nun da stimmt einiges nicht. Das Universum schrumpft nicht zu einem Punkt zusammen. Denn nur die Ausdehnung in der Bewegungsrichtung macht dies. Ebenso werden zwei Ereignisse an einem Ort des Universums für dieses “Teilchen” in einer unendlichen Zeit vergehen.

    Und dann sollte man bedenken, daß wegen des Wellencharakters des Lichtes dieses Teilchen im Prinzip überall ist.

    Man kann wohl solche Spekulationen machen aber wir sollten doch eher uns um Physik als um Metaphysik unterhalten. Der Welle-Teilchen Dualismus wie das Relativitätsprinzip scheinen zwar widersprüchlich zu sein aber man kann “wirkliche” Aussagen für ein Experiment oder ein Gerät machen.

  41. Hallo Herr Knoth,

    “Nun da stimmt einiges nicht. Das Universum schrumpft nicht zu einem Punkt zusammen. Denn nur die Ausdehnung in der Bewegungsrichtung macht dies. Ebenso werden zwei Ereignisse an einem Ort des Universums für dieses “Teilchen” in einer unendlichen Zeit vergehen.”

    Könnten Sie dies genauer erklären – ich meine, was da nicht stimmt.

    Das Etwas, was wir Strahlung nennen, bewegt sich in eine Richtung ? Ich dachte immer, der Energieübergang in einem Atom erzeugt ein kugelsymmetrisches Etwas. Hat diese Kugelsymmetrie nun doch eine Richtung, ja wie geht das denn ?

    Und welche unendliche Zeit meinen Sie. Die Eigenzeit ist in dem bewegten Bezugssystem ist definitiv Null. Ein Beobachter, der im Bezugssystems der Strahlung mit der Strahlung bewegt, sieht die Stahlung im gleichen Moment entstehen und vergehen. Aus dem genannten Ruhsystem (unser Beobachtungspunkt) betrachtet natürlich nicht, da braucht das Licht von der Sonne zur Erde glaube ich 7 Minuten.
    Wenn Stahlung aber kein Bezugssystem hat, bzw. dieses zu konstruieren nicht erlaubt ist, woher hat Strahlung dann einen Impuls ? Impuls ist eine Größe, deren Wert bezugssystemabhängig ist, es sein denn, der Impuls von Strahlung ist kein “normaler” Impuls, aber worin bitte genau unterscheidet er sich dann inhaltlich vom normalen Impuls ?

    Grüße Fossilium

  42. @fossilium 29. Juni 2018 @ 15:30

    Das Etwas, was wir Strahlung nennen, bewegt sich in eine Richtung ? Ich dachte immer, der Energieübergang in einem Atom erzeugt ein kugelsymmetrisches Etwas. Hat diese Kugelsymmetrie nun doch eine Richtung, ja wie geht das denn ?

    Nun Sie sprachen von einem Photon mit einer definierten Bewegungsrichtung. Und jetzt nicht mehr?

    Und welche unendliche Zeit meinen Sie. Die Eigenzeit ist in dem bewegten Bezugssystem ist definitiv Null. Ein Beobachter, der im Bezugssystems der Strahlung mit der Strahlung bewegt, sieht die Stahlung im gleichen Moment entstehen und vergehen. Aus dem genannten Ruhsystem (unser Beobachtungspunkt) betrachtet natürlich nicht, da braucht das Licht von der Sonne zur Erde glaube ich 7 Minuten.

    Nun ja das ist das Seltsame mit der Relativität. Die Erde bewegt sich dann auch mit Lichtgeschwindigkeit auf dieses Teilchen. Also dauert die Zeit zwischen zwei Ereignissen auf der Erde in diesem Bezugsystem unendlich lang.

    Wenn Stahlung aber kein Bezugssystem hat, bzw. dieses zu konstruieren nicht erlaubt ist, woher hat Strahlung dann einen Impuls ?

    Nun jetzt wechseln Sie wieder von Relativitätstheorie zu Quantentheorie. Für den Impuls braucht es kein Bezugsystem.

    Ich denke wir bleiben lieber bei der Relativitätstheorie und wechseln nicht zwischen beiden Theorien hin und her. Oder lassen erstmal solche exotischen Fälle wie Bezugsystem eines Photons aussen vor.

  43. Lieber Herr Knoth,

    Strahlung hat einen mechanischen Impuls, der sich im Grenzübergang v nach c aus der Energie-Impuls Beziehung E=p*c ergibt. Und diesen Impuls hat Strahlung ganz gleich, ob man Strahlung als Schauer von Photonen oder als Welle ansieht. Das alles hat mit Quantenphysik überhaupt nichts zu tun. Wenn sich die Erde mit Lichtgeschwindikeit auf das Teilchen zubewegt, ist auch die Eigenzeit auf der Erde null (gegenüber dem Teilchen als Ruhsystem).

    Nun gut, Sie wissen die Antworten nicht, vielleicht ein anderer Physiker ?

    Grüße Fossilium

  44. Zitat Frau Lopez 28.06.18@22:11

    “Die hochtrabende Geschwindigkeitsaddition Einsteins enthält zwar ausdrücklich den
    Term c + v, jedoch ist v dabei immer als v = 0 vorausgesetzt, keine andere Größe!”

    Nirgend setzt Einstein v=0 voraus, der Term (c+v) steht nicht allein, da steht ein Bruch!

    Original Einstein 1905 Seite 906 für das Additionstheorem U=v⊕w , also die
    relativistische Geschwindigkeitssumme U=(v+w)/(1+vw/V²) wobei V=c bedeutet.
    Für Krückstöcke http://users.physik.fu-berlin.de/~kleinert/files/1905_17_891-921.pdf
    Wird nun für eine der beiden Geschwindigkeiten maxV gesetzt, Einstein hat v=V=c
    genommen, dann gilt für beliebige w nicht nur für w=0 U=V=c.
    Man hat doch in der Schule Bruchrechnung gehabt, der Nenner gehört dazu 11elf!
    Das ist mit banaler Schulmathematik irgendwo Klasse 8 zu rechnen, aber L.
    1. liest wohl nicht richtig (?) oder mißt dem Nenner nicht seine Bedeutung zu
    2. zitiert falsch, indem sie dann den Nenner in ihrer Privatmathematik unterschlägt
    3 zieht falsche Schlußfolgerungen und schiebt Einstein ihren Fehler in die Schuhe

    Richtiges Ergebnis “Der Nenner ist immer größer 1 und kompensiert den Zähler (c+v) so,
    daß für beliebige v die Summe (c+v) nicht größer c also LG werden kann.”

    Praktisches Beispiel: ein Stern sende grünes Licht aus und 3 Beobachter.
    – Beobachter 1 ruht zum Stern und sieht erwartungsgemäß grünes Licht und mißt c
    – Beobachter 2 fliegt zum Stern, sieht aber blaues Licht und mißt auch c
    – Beobachter 3 fliegt weg, sieht dafür rotes Licht und mißt wieder c
    Nicht die LG ändert sich c=const, Farbe des Lichtes ändert sich – wollte die Natur so.

  45. @fossilium 29. Juni 2018 @ 16:41

    Was soll diese Diskussion? Ich habe 2 Punkte erklärt:

    1. Den exotischen Grenzfall v=c lassen wir beiseite.
    2. Kein Wechsel zwischen Teilchen und Welle bei Licht.

    Das alles hat mit Quantenphysik überhaupt nichts zu tun. Wenn sich die Erde mit Lichtgeschwindikeit auf das Teilchen zubewegt, ist auch die Eigenzeit auf der Erde null (gegenüber dem Teilchen als Ruhsystem).

    Und da haben wir wieder diese schöner Vermischung zwischen Teilchen und Welle. Sie stellen die Photonen als lokalisierbare Teilchen dar. Dies ist Unsinn. I)ch hatte an einem Experiment mitgearbeitet, in dem Gammaquanten von Kristallen reflektiert wurden. Diese Teilchen konnten gezählt werden, aber sie wurden nach Beugungsgesetzen reflektiert.

    Ich habe den Eindruck, daß Sie hier Verwirrung stiften wollen. Ein Ruhesystem des Lichtes gibt es nicht. In allen Inertialsystemen hat dies die Geschwindigkeit Null. Und ein massereicher Körper wir die Erde daher nie eine Relativgeschwindigkeit c.

  46. Korrektur zum letzten Kommentar:

    Ich habe den Eindruck, daß Sie hier Verwirrung stiften wollen. Ein Ruhesystem des Lichtes gibt es nicht. In allen Inertialsystemen hat dies die Geschwindigkeit Null. Und ein massereicher Körper wir die Erde daher nie eine Relativgeschwindigkeit c.

    Gemeint ist: “In allen Inertialsystemen hat das Licht die Geschwindigkeit c Und ein massereicher Körper wie die Erde daher nie eine Relativgeschwindigkeit c”.

  47. Hi zusammen,

    ich muß noch mal nachfassen: das Thema hier lautet: mechanische Systeme und Koordinatensysteme, und da ist – so glaube ich – keinesfalls alles so klar (jedenfalls mir nicht), wie es Herr Schulz behauptet. ich hoffe ich krieg es verständlich hin und es wird nicht zu lang:

    Die SRT sagt uns, es gibt kein Inertialsystem, das vor allen anderen ausgezeichnet ist, und die Lichtgeschwindigkeit ist in allen Inertialsystemen immer gleich groß. Das trifft wohl auch zu, außer für ein Bezugssystem, daß sich mit einem Objekt mitbewegt, das sich selbst mit Lichtgeschwindigkeit bewegt. Sollte es so eines geben, wäre dieses ausgezeichnet, weil die Lichtgeschwindigkeit in diesem System – im Gegensatz zu allen anderen – definitionsgemäß den Betrag Null hätte. Also muß die Behauptung, es gäbe kein besonderes Intertialsystem, und die Lichtgeschwindigkeit wäre in allen System gleich, einschränken auf a l l e Systeme – a u ß e r einem, daß sich selbst mit Lichtgeschwindigkeit bewegt. Es sein denn, dieses würde nicht existieren, dann kann man die Bezeichnung “alle” lassen. Das ganze ist nicht nur Wortklauberei, denn :

    Offenbar kann man jedem bewegten physikal. Objekt eine Geschwindigkeit, eine Bewegungsenergie, und einen Impuls zuordnen. Diese drei Größen dienen u.a. dazu, den Bewegungszustand des Objektes vom Ruhezustand zu unterscheiden, d.h. es sind relative Größen (Beträge sind Differenzen), und sie sind vom Bezugssystem abhängig: bezgl. mitbewegter Systeme sind die Objekte in Ruhezustand, haben dann z.B. Null Geschwindigkeit, Null Impuls und Energie, bezgl. eines anderen Systems kann man Ihnen jeweils einen Wert zuordnen.

    Hierbei wird allerdings stillschweigend die Voraussetzung gemacht, daß sich das Objekt, egal welches, auf dem kürzesten Weg von a nach b macht, um die Größe „Geschwindigkeit“ überhaupt sinnvoll zu definieren. Im Fall der Strahlung wissen wir nicht, was für ein Objekt ontologisch da vorhanden ist, im Bild einer elm. Welle bewegt sich z.B. alles durch den ganzen Raum. Ich will hier aber nicht drauf herumreiten, wir tun jetzt mal pragmatisch so, als hätten wir auch im Fall von Strahlung so was wie eine „Front“, die sich gradlinig fortpflanzt, wie auch immer, und daß auch für Strahlung so etwas wie eine Geschwindigkeit, nämlich die Lichtgeschwindigkeit vom Betrag c, definieren läßt (jemandem wie mir sträuben sich die Nackenhaare, aber egal).

    Mit diesem pragmatischen Zugeständnis ist es dann tatsächlich so, daß man jedem Bewegungszustand – egal aus welchem Bezugssystem heraus dieser beschreiben wird (mit der obigen Ausnahme) – drei Werte zuordnen kann, einen für die Geschwindigkeit, einen für die Bewegungsenergie und einen für den Impulsbetrag, und daß dies auch gilt für Bewegungszustände, die sich mit Lichtgeschwindigkeit bewegen. Solche Lichtgeschwindigkeits-Bewegungszustände sind aber insofern ausgezeichnet, daß sich die drei Werte (für Geschwindigkeit, Energie und Impuls) beim Wechsel von einem in ein anderes Bezusgsystem nicht ändern (sind galilei- und lorenzinvariant), den c ist immer in allen Bezugssystemen gleich groß. Aber daß ein Lichtgeschwindigkeits-Bewegungszustand was besonderes ist, das kann man akzeptieren, auch wenn mit diesem verknüpft plötzlich galilei- und lorenzinvariante Energien und Impulse auftreten. Oder nicht ? Jetzt wird mir mulmig.

    Aber weiter: die Bewegungsenergie und der Impuls sind nun mal der Geschwindigkeit proportional (ist logisch): E prop v und p prop v bei Bewegungszuständen mit Geschwindigkeiten v < c, und E prop c und p prop c bei solchen, die sich mit Lichtgeschwindigkeit bewegen. Woher nehme ich den Proportionalitätsfaktor ? Bei Bewegungszuständen mit v < c haben die Objekte eine träge Masse, und dann kann ich – vereinfacht gesagt – diese als Proportionalitätsfaktor für den Impuls hernehmen: p = m*v, und den Impuls als Proportionalitätsfaktor für die Bewegungsenergie: E = p*V (immer bis auf dim.lose Faktoren). Aber bei Bewegungszuständen, die sich mit Lichtgeschwindigkeit bewegen, habe ich keine träge Masse. Wie kann ich für diese Bewegungszustände einen definierten Impuls bilden ? Es gibt keine theoretischen oder metaphysischen Überlegungen, die mir den Proportionalitätsfaktor liefern könnten.

    Deswegen verfalle ich auf einen Trick: ich setze einfach (formal, ohne inhaltliche Klarheit !) p = E/c und E = P*c – schwupp, und schon habe ich eine definierte Energie und einen definierten Impuls, metaphysisch zwar im Zirkelschluß geschaffen – aber was solls: beide Größen stehen jetzt für einen Bewegungszustand, der sich mit Lichtgeschwindigkeit bewegt, zur Verfügung, was diese jetzt inhaltlich bedeuten, wenn sie so gebildet werden, ist mir zwar schleierhaft – aber los geht’s, mal sehen ob es klappt ?

    Und es klappt!

    Es war natürlich der Anspruch von Herrn Schulz zu zeigen, daß in der speziellen Relativitätstheorie der Begriff des Bezugssystems und die Theorie insgesamt in sich geschlossen, logisch klar und keinerlei Widersprüche aufweist und sich gegen Einwände unverbesserlicher Widersacher mit restloser Klarheit leicht verteidigen läßt. Mit den vorstehenden Ausführungen könnte sich aber herausgestellt haben, daß auch in der SRT pragmatisch mit den Begriffen umgegangen wird („alle“ Inertialsystem), ja, daß man auch vor Tricks nicht zurückschreckt, um alles stimmig machen. Ich meine, es ist letzlich bewundernswert, wie es die Physik schafft, inhaltliche schwere Hindernisse mit Raffinesse und Nonchalance zu überwinden – mit dem Wort „Klarheit“ würde ich aber etwas anderes bezeichnen.

    Letztlich tut das meiner Bewunderung für die SRT keinen Abbruch. Aber manches funktioniert eben wie überall in der Physik nur durch Tricks und Schummelei (sehr geschickt), und total durchdacht und so gut verstanden, wie es Herr Schulz meint, ist es dann eben nicht ( trotzdem gut).

    Grüße Fossilium

  48. @fossilium 30. Juni 2018 @ 16:32

    Nun da jetzt endlich klar ist, worauf Sie hinauswollen, mein Kommentar dazu.

    Das trifft wohl auch zu, außer für ein Bezugssystem, daß sich mit einem Objekt mitbewegt, das sich selbst mit Lichtgeschwindigkeit bewegt. Sollte es so eines geben, wäre dieses ausgezeichnet, weil die Lichtgeschwindigkeit in diesem System – im Gegensatz zu allen anderen – definitionsgemäß den Betrag Null hätte.

    Nun was für ein Objekt soll denn das sein? Es darf keine Masse haben, weil es sonst nie die Lichtgeschwindigkeit erreicht. Das kann nur die Lichtwelle (etwa ein Wellenkamm) oder ein Photon sein. Den Wellenkamm kann man aber nciht in diesem Bezugsystem identifizieren, weil die Frequenz und damit die Wellenlänge nach dem Dopplereffekt Null b.z.w. unendlich sind. Das Photon hat die Ruhemasse Null. Im Artikel steht aber ein Objekt mit Ruhemasse grösser Null. Und von einem Photon oder einer EM-Welle geht auch kein Licht aus.

    Mit den vorstehenden Ausführungen könnte sich aber herausgestellt haben, daß auch in der SRT pragmatisch mit den Begriffen umgegangen wird („alle“ Inertialsystem), ja, daß man auch vor Tricks nicht zurückschreckt, um alles stimmig machen. Ich meine, es ist letzlich bewundernswert, wie es die Physik schafft, inhaltliche schwere Hindernisse mit Raffinesse und Nonchalance zu überwinden – mit dem Wort „Klarheit“ würde ich aber etwas anderes bezeichnen.

    Nun ob die Worte Trick oder Raffinesse hier sinnvoll, kann man diskutieren. Aber Sie wenden selbst den Trick an, einem masselosen Objekt oder einer unendlich langen Welle ein Inertialsystem als Bezugspunkt zuzuordnen.

  49. Hallo Herr Knoth,

    ich kann mir ein Koordinatensystem, das sich mit Lichtgeschwindigkeit durch den Raum bewegt, durchaus vorstellen, es wird sich von einem solchen mit v < c gar nicht unterscheiden. Ich kann mir sogar vorstellen, daß es sich – von einem ruhenden Koordinatensystem aus, also von außen, betrachtet – sich in Richtung seiner x-Achse bewegt, während Feldstärkevektoren längs seiner y-Achse periodische Schwankungen durchführen, daß sich sogar die Frequenz dieser Schwankungen abrupt ändert, weil plötzlich ein Elektron durch das Koordinatensystem geflogen ist.

    Also dass das, was man Strahlung nennt, von der Sonne zur Erde fliegt, und dabei von einem Koordinatensystem gleicher Geschwindigkeit begleitet wird, das ist sehr gut vorstellbar, aber seine Existenz ist eben von der SRT verboten, denn die Geschwindigkeit der Strahlung, die da fliegt, wäre ja, von diesem Koordinatensystem aus gemessen, gleich Null – und damit wäre dieses System ausgezeichnet gegenüber allen anderen, in denen c = const = zwingend ungleich Null ist, was nicht sein darf.

    Natürlich wären von diesem mit c bewegten Koordinatensystem aus gemessen alle Längen = Null und die Eigenzeit wäre auch Null – es würde darin also überhaupt nichts geben oder passieren, sodaß man von daher sagen kann, so was kann man zwar als Koordinatensystem sich vorstellen, aber von da aus ist kein physikalischer Prozess zu beobachten und deshalb würde so ein System keinen Sinn machen. Das ist logisch konseqent, wenn man es so betrachtet.

    Das Ganze hat aber etwas Absurdes (ich hab früher erst mal singulär gesagt). Die gesamte Energie, die von außen auf die Erde kommt, wird von einem Medium transportiert, das man in einem Koordinatensystem anordnen kann, welches keine Perspektive nach außen ermöglicht. Man fragt sich doch da, ob sich hier überhaupt etwas durch den Raum bewegt, und ob man den Begriff Geschwindigkeit auf sowas, was zu allem Überfluß auch nicht mal das Geschwindigkeits-Additionstheorem erfüllt, überhaupt anwenden kann, und ob die sog.Lichtgeschwindigkeit nicht einfach nur formal eine Geschwindigkeit ist, konzeptionell bzw. inhaltlich metaphysich aber was ganz anderes. Und man fragt sich, wie dieses Medium auch noch eine klassische Energie- und einen klassischen Impuls mit sich führt, die man beide zwar mit einem Taschenspielertrick definiert, die dann aber auch noch mit den experimentellen Ergebnissen übereinstimmen.

    Die SRT sagt, alles ist relativ, nur eines nicht, nämlich die Geschwindigkeit der Strahlung, na ja, das so was wie: alles ist Regel, aber es gibt eine Ausnahme – ich meine, das ist kein Konzept für eine endgültige Theorie. Eine solche würde erklären, wo Raum und Zeit und ein konstantes c herkommen, also harmonischer sein, als eine Regel mit Ausnahme. Bei all den Ungereimtheiten frage ich mich, wie jemand behaupten kann, er habe die SRT "voll" verstanden und sie mit Zähnen und Klauen auf Teufel komm raus (z.B. Beitrag löschen) gegen jeden noch so kleinen Widerspruch verteidigt, als ginge es um Leben und Tod (der Physik).

    ich meine, einfach cool bleiben, und auch Skeptiker und Kritiker mal verstehen, wäre nicht das Schlechteste. Herrn Schulz will ich da übrigens ausnehmen, da ist schon einige Nachdenklichkeit da (aus meiner Sicht nicht genug, aber bißchen ist auch schon viel) – aber nicht alles an der Physik immer nur wunderbar zu finden würde anderen in diesem Forum ganz gut anstehen.

    Grüße Fossilium

  50. Hallo fossilium, in der “normalen” Raumzeit funktionieren deine Wünsche eher nicht so.
    Aber was du willst, hat Ähnlichkeiten mit dem Wasserfallmodel über den Ereignishorizont
    eines Schwarzen Loches. Der Raum stürzt dann nach der “Klippe” mit Über-LG auf die
    Singularität zu, innerhalb der “Raumelemente” gilt die SRT mit c=const.
    Grüße Dip

  51. @fossilium 30. Juni 2018 @ 22:03

    Nun da Sie sich doch recht ausführlich ausgelassen haben, gebe ich noch einen (hoffentlich) letzten Kommentar ab.

    Also dass das, was man Strahlung nennt, von der Sonne zur Erde fliegt, und dabei von einem Koordinatensystem gleicher Geschwindigkeit begleitet wird, das ist sehr gut vorstellbar, aber seine Existenz ist eben von der SRT verboten, denn die Geschwindigkeit der Strahlung, die da fliegt, wäre ja, von diesem Koordinatensystem aus gemessen, gleich Null – und damit wäre dieses System ausgezeichnet gegenüber allen anderen, in denen c = const = zwingend ungleich Null ist, was nicht sein darf.

    Natürlich kann man sich im Prinzip auch ein Koordinatensystem vorstellen, das sich mit Licht bewegt. Man sollte bedenken, dass die Bezugsobjekte (gern auch Beobachter genannt) folgende Eigenschaften haben:

    Es muss möglich sein, Messungen mit Licht durchzuführen. Dies betrifft die im Koordinatensystem ruhende Objekte, wie auch Objekte, die sich relativ zu diesem Koordinatensystem bewegen. Diese müssen daher Licht senden und empfangen können. Dies würde aber meines Wissens Objekte mit Ruhemasse > 0 voraussetzen. Diese gibt es aber nicht.

    Man fragt sich doch da, ob sich hier

    überhaupt etwas durch den Raum bewegt,

    und ob man den Begriff Geschwindigkeit auf sowas, was zu allem Überfluß auch nicht mal das Geschwindigkeits-Additionstheorem erfüllt, überhaupt anwenden kann, und ob die sog.Lichtgeschwindigkeit nicht einfach nur formal eine Geschwindigkeit ist, konzeptionell bzw. inhaltlich metaphysich aber was ganz anderes.

    Nun ein langer Satz mit mit mehreren Fehlern:

    1.

    überhaupt etwas durch den Raum bewegt,

    Welchen Raum denn? Den in dessen Inertialsystem die Sonne ruht, oder das Zentrum der Milchstrasse?

    2.

    Geschwindigkeit auf sowas, was zu allem Überfluß auch nicht mal das Geschwindigkeits-Additionstheorem erfüllt,

    Wie dass denn? Man kann das Additionstheorem auch auf eine der Geschwindigkeiten = c anwenden und erhält dann c.

    Die SRT sagt, alles ist relativ, nur eines nicht, nämlich die Geschwindigkeit der Strahlung, na ja, das so was wie: alles ist Regel, aber es gibt eine Ausnahme – ich meine, das ist kein Konzept für eine endgültige Theorie. Eine solche würde erklären, wo Raum und Zeit und ein konstantes c herkommen, also harmonischer sein, als eine Regel mit Ausnahme.

    Nun die Aussage “alles ist relativ” ist eine starke Vereinfachung der Aussagen der SRT. Zwei Beobachter, die in jeweils sich zueinander bewegenden Koordiantensystemen ruhen, stimmen auf jeden Fall über die Lichtgeschwindigkeit und die Geschwindigkeit, mit der sich die Koordinatensyteme bewegen überein. Nur Raum und Zeit sind nicht mehr absolut. Daraus folgen vor allem für die Zeit Konsequenzen wie die “Relativität der Gleichzeitigkeit” und die Zeitdilatation. Aber einen allgemeinen “Relativismus” beinhaltet die SRT nicht.

    Bei all den Ungereimtheiten frage ich mich, wie jemand behaupten kann, er habe die SRT “voll” verstanden und sie mit Zähnen und Klauen auf Teufel komm raus (z.B. Beitrag löschen) gegen jeden noch so kleinen Widerspruch verteidigt, als ginge es um Leben und Tod (der Physik).

    Ich denke, jetzt kommen wir zum Kern der Sache. Erstmal wäre es mal interessant zu wissen, wen Sie meinen und welche Beiträge zu Unrecht gelöscht wurden. Und der Begriff “jeden noch so kleinen Widerspruch” ist bei der Fundamentalopposition gegen die SRT eine Untertreibung. Wenn Diskussionsteilnehmer (Kritiker) selbst die GPS-Spezifikation als Lüge bezeichnen, dann ist das mehr als ein “kleiner Widerspruch” sondern die totale Ablehnung der SRT.

    Und wenn Sie hier Herrn Pössel meinen, dann hat dieser meiner Ansicht nach eine grosse Geduld aufgebracht, um das Thema “Relativität der Gleichzeitigkeit” zu diskutieren und sogar für Herrn Dr. Engelhardt einen Beitrag zu diesem Thema geschrieben. Nur wenn es zu diesem Thema dann fast 2000 Kommentare gibt, und die “Kritiker” immer noch nicht einlenken wollen, dann ist seine Reaktion “verständlich”. Herr Schulz ist übrigens auch kein Freund von wochenlangen Diskussionen.

  52. @fossilium

    Deswegen verfalle ich auf einen Trick: ich setze einfach (formal, ohne inhaltliche Klarheit !) p = E/c und E = P*c – schwupp, und schon habe ich eine definierte Energie und einen definierten Impuls, metaphysisch zwar im Zirkelschluß geschaffen

    E=m*c²/sqrt(1-v²/c²) → E²/(m²*c^4)=1/(1-v²/c²)
    p=m*v/sqrt(1-v²/c²) →1+p²/(m²*c²)=1/(1-v²/c²)

    Verbindet man beide Gleichungen miteinander, indem man die rechten Seiten eilminiert, so erhält man:

    m²=E²/c^4-p²/c² und speziell für m=0 gilt dann: p=E/c

    Worin liegt nun der Zirkelschluss? Das einzige was mich da ein wenig stutzig macht ist, dass man bei Photonen folgende Gleichung erhält:

    E=0/0 und p=0/0

    Wie viel ist beispielsweise f(x)=sin(x)/x.
    Die Gleichung in den Plotter eingegeben, würde ich spontan behaupten: f(0)=1.
    Ist das in dem Fall mathematisch korrekt?

  53. Fossiliums Fragen zum Koordinatensystem mit Lichgeschwindigkeit lassen sich leicht beantworten:
    Ein Koordinatensystem, dass sich mit einer elektromagnetischen Welle mitbewegt, ist leicht zu konstruieren. Zum beispiel durch Galilei-Transformation aus einem Inertialsystem. Es lässt sich aber leicht sehen , dass das kein Inertialsystem sein kann: Wir haben dann nämlich ein Koordinatensystem geschaffen, in dem eine elektromagnetische Welle einfach so im Raum steht. Das widerspricht aber den Maxwell-Gleichungen, nach dem ein elektrisches Feld stets eine Änderung des Magnetfelds induziert und damit solch eine Welle nicht statisch sein kann. Da nun ein Inertialystem definiert ist als eines, in dem die Naturgesetze gelten wie in Ruhe, ist solch ein Koordinatensystem kein Inertialsystem.

    Eine Theorie, die Elektromagnetismus einschließt, darf also keine Vorschrift angeben, ein Inertialsystem zu konstruieren, in dem eine Lichtwelle steht. Die Lorentztransformation genügt dieser Anforderung: Beim Versuch, Lichtgeschwindigkeit in die Transformationsformel einzugeben, erhält man eine Division durch Null. Und das gibt nicht unendlich, es ist schlicht nicht zulässig.

    Es gibt kein Ruhesystem des Lichts, denn ein Ruhesystem ist definiert als ein Inertialsystem, in dem das Objekt ruht. Irgendwelche Koordinaten, in denen Licht ruht, sind leicht zu konstruieren.

  54. Hallo Herr Schulz,
    die Fragen zum Koordinatensystem bzw. Inertialsystem sind auch direkt nicht das Problem, die SRT ist sicherlich eine sehr konsistente, in sich geschlossene Theorie.

    Trotzdem schleppt sie ein Problem mit sich herum:

    Das Problem ist der Impuls der Strahlung – zu aller Überraschung ein tatsächlich vorhandener meßbarer (!) Impuls, d.h. diesen gibt ja wirklich, er ist nicht nur formale Rechengröße.

    Der Impuls ist eine Größe, die ich einem Bewegungszustand zuordnen kann. Dazu brauche aber etwas, was in diesem Zustand ist. In der Mechanik ist dies eine Masse. Was ist dies bei der Strahlung ? Vor allem muß der Impuls der Strahlung irgendwo herkommen ? Dass man der Strahlung Bewegungsenergie zuordnen kann, das ist klar, weil sich die Energie des emittierenden Atoms ja verringert – aber wie entsteht der Impuls ? Entsteht der aus dem Nichts, indem plötzlich ein positiver und ein negativer Impulsvektor da ist, und das Atom eine Art Rückstoß bekommt ? Oder kommt der Impuls aus der Atombewegung ? Da ja modellhaft kugelförmige Strahlungsfronten entstehen, müßten diese Fronten alle kleine Impulspfeile haben, die senkrecht darauf stehen, diese würden sich alle in der Summe auf Null addieren, ebenso wie die Rückstoßpfeile. Der Gesamtimpuls wäre dann null, aber jede Strahlungsfrontausschnitt hat immer noch einen Impuls(pfeil) – und der wird auch noch gemessen. Mechanisch kann man das wohl so nicht erklären, aber wie dann ? Bitte nicht mit der Quantenmechnik kommen : der quantenmechanische Impuls und die Energie hängen über das Wirkungsquantum zusammen und beschreiben primär keine Bewegungszustände.

    Vielleicht kennzeichnen Bewegungsenergie und Impuls auch bei der Strahlung gar nicht den Bewegungszustand – aber was kennzeichnen sie dann ? Wieso messe ich, wenn sie den Bewegungszustand nicht kennzeichnen, sondern was anderes, dann einen klassischen Impuls ?

    Und das zweite ist doch: ich kann mir Bewegungszustände von irgendwas vorstellen, die mit immer größerer Geschwindigkeit ablaufen (Cern-Röhre), und ist steigere die Geschwindigkeit immer mehr, ich komme immer näher an die Grenze c, und an dieser Grenze habe ich plötzlich so eine Art “Unstetigkeit”: “Massen” werden unendlich, Geschwindigkeiten knicken ab auf einen konstanten Wert, kein Inertialsystem mehr, usw. – aber was bleibt ist ein stetiger Übergang bei Energie und Impuls, kein Problem mit diesen beiden Größen, im Grenzübergang v nach c bleiben diese beiden definiert – und lassen sich auch noch messen, also nicht nur formale Definition, sondern vorhandene Werte !

    Deshalb meine ich, hier ist noch etwas nicht verstanden – was aber der SRT ihren Glanz keinesfalls wegnimmt, keine Theorie ist ohne Grenzen. Vielleicht haben Sie (oder Herr Senf) eine Erklärung oder Anmerkung zu diesem Phänomen Impuls.

    Grüße Fossilium

  55. Ach so, und was mir noch einfällt: hat das Neutrino nicht eine Masse und bewegt sich doch mit Lichtgeschwindigkeit.
    Grüße Fossilium

  56. Über allem schwebt immer noch das metaphysische Problem der Geschwindigkeit an sich: jede Definition setzt voraus, daß vom Objekt, dem man den Geschwindigkeitsbetrag c = dx / dt zuordnet, dx auf kürzestem Weg durchlaufen wird. Bei der Strahlung habe ich keinen Weg, daher auch keinen kürzesten. ich hab nur modellhaft einen Weg (Strahlungsfront), c ist daher nur modellhaft definiert. Trotzdem meßbar. Das paßt nicht zusammen. Die ganze SRT wäre unter diesem Blickwinkel auf einem fraglichen Begriff c aufgebaut, der sich als etwas ganz anderes herausstellen könnte. Ich will hier aber nicht rumunken und das Thema begraben. Schließlich ist, wenn man es nur will, alles relativ und von verschiedenen Seiten betrachtbar, und neben den glanzvollsten Leuchttürmen gähnen manchmal tiefste Abgründe. Da muß man nicht reinschauen (die schauen sonst zurück).

  57. @fossilium

    Ach so, und was mir noch einfällt: hat das Neutrino nicht eine Masse und bewegt sich doch mit Lichtgeschwindigkeit.
    Grüße Fossilium

    Nein nur sehr nahe an der Lichtgeschwindigkeit. Nur den Unterschied kann man schwer messen. In der Betrachtung zum Beta-Zerfall wurde im Kernphysik-Praktikum wirklich für eine einfachere Rechnung die Masse gleich Null gesetzt.

    Bei der Strahlung habe ich keinen Weg, daher auch keinen kürzesten. ich hab nur modellhaft einen Weg (Strahlungsfront), c ist daher nur modellhaft definiert.

    Nein das Fermatsche Prinzip spricht vom zeitlich kürzesten Weg. Oder wie Peilen Sie ein Objekt an?

  58. @fossilium
    Es würde wirklich helfen, wenn Sie wirklich sauber argumentieren würden. Stattdessen wechseln Sie munter Ihre Position, wie es Ihnen gefällt.

    Zum Impuls: Natürlich gibt es einen Impuls. Und der entsteht nicht aus dem Nichts, sondern bei der Aussendung eines Photons in eine Richtung gibt es in der Tat einen Rückstoß – nur ist der Impuls des Photons normalerweise so gering, dass die Geschwindigkeit, die das Atom durch den Rückstoß erreicht, winzig ist. Und eine Strahlungsquelle strahlt im Allgemeinen in alle Richtungen, so dass die Rückstöße sich aufheben.

    Bei der Geschwindigkeit haben Sie in jedem existierenden Inertialsystem immer noch einen Weg und eine Zeit, die Sie messen könnten, nur in einem nicht existierenden Inertialsystem, in dem Licht in Ruhe wäre, würden Sie beide verlieren.

    Und zu den Neutrinos: Die haben eine wirklich winzige Ruhemasse, bei ihrer Entstehung bekommen sie aber viel mehr Energie, als es dieser Ruhemasse entspricht. Die gesamte restliche Energie äußert sich als Geschwindigkeit – erst bei Geschwindigkeiten, die sich fast nicht von der des Lichts unterscheiden, erreicht die relativistische Masse des Neutrinos einen Wert, die der Energie entspricht, die das Neutrino bei seiner Entstehung mitbekommen hat.

  59. @Joachim Schulz
    An sich war ich heute hergekommen,um Sie zu bitten, eine Ihrer Äußerungen zu korrigieren bzw präzisieren. Aber wenn ein Chor der Einsteinleugner jede solche Korrektur sofort als Beweis der Falschheit der Relativitätstheorie auslegen würde, gibt es dafür wohl keine Chance.

    Ich werde es wohl nicht mehr erleben, dass die Einsteinleugner entweder verschwinden oder so verlacht werden, wie sie es verdienen, und endlich wieder eine rationale Diskussion möglich wird.

  60. Hallo fossilium, wenn du es mechanisch sehen möchtest,
    bist du bestimmt einverstanden, daß ein Impuls da ist, wenn eine Kraft auftritt.
    Eine elektromagnetische Welle hat eine elektrische und eine magnetische Feldstärke.
    Beide üben zB auf ein Elektron eine Kraft aus, also hat die Welle Impuls. Die Energie
    der Welle leistet Arbeit am Elektron, die bestimmt den Impuls und den Strahlungsdruck.

  61. @Frank Schmidt heute 20:18

    Nun haben sich als “Einsteinleugner” nur Frau Lopez und “fossilium” hervorgetan. Eventuell können Sie Ihre Frage nochmal stellen.

  62. Hi Rudi Knoth,
    Quatsch, ich bin doch kein Einsteinleugner. Herr Schmidt versteht bloß die Physik nicht, weil bei ihm zwei Impulse aus dem Nichts enstehen, und sich die erzeugten Impulse in der einen Richtung aufheben, die gleich großen in der entgegengesetzten Richtung aber nicht. Und lieber, Herr Senf, Ihre Rede geht an der Frage vorbei: woher kommt der Impuls der Strahlung, bestimmt nicht daher, daß die Strahlung einem Elektron einen Impuls verleihen kann. Dies bestätigt ja nur, daß Strahlung einen Impuls hat, der zum Teil übertragen wird, ich habe aber gefragt, wo kommt der Impuls der Strahlung her, wie geht er bei der Entstehung (?) auf die Strahlung über, sodaß sie ihn besitzt , bzw. sodaß man der Strahlung einen Impuls als Eigenschaft zuorden kann. Welche Kraft soll denn diesen derStrahlung verliehen haben, und mit welcher Gleichung kann ich aus dieser ominösen Kraft den gemessenen Impuls herleiten. Kräfte wirken zudem nur lokal, Strahlung ist nicht lokal – oder wird bei der Emission aus was Lokalem was Raumgreifendes, und welche Kräfte sollen das sein ?

    Es ist mir egal, ich will es gar nicht wissen, ich will nur etwas verstehen – und so sehr ich mich anstrenge, es ist ein Punkt, der bei der SRT (von mir jedenfalls) nicht verstanden ist. Das gilt auch für die Größe c. Jetzt möchte ich gerne wissen ob die SRT das Prinzip der kleinsten Wirkung voraussetzt. Ich glaube nicht, eher setzt das Fermatsche Prinzip die Konstanz der Lichtgeschwindigkeit voraus. Es könnte sogar sein, dass es selbst voraussetzt, das die Geschwindigkeit immer über den kürzesten Weg zwischen zwei Punkten definiert ist, und von daher diese eigene Voraussetzung im Zirkelschluß ableitet. Das wäre ja ein Ding, wenn daß bisher keinem (auch mir nicht) aufgefallen ist.

    Grüße Fossilium

  63. Also das massebehaftete Neutrino hat keine Lichtgeschwindigkeit – kann ja auch nicht anders sein, würde der SRT widersprechen. Und es fliegtmit winziger Masse immer auf dem kürzesten Weg von a nach b mit einer Geschwindigkeit innerhalb enger Grenzen angenähert an c. Falls nicht würde c = constant keinen Sinn machen. Damit hat es den Charakter eines (teilchenhaften) Quantenobjektes auf einer Bahn. Ist so eine System- konfiguration für ein Einteilchensystem gem. der relativ. Quantentheorie einerseits und den Heisenbergschen Unschärferelationen andererseits erlaubt – oder sind die Umschärfen im zulässigen Bereich ?

  64. Ach so, als ich fragte, haben Neutrinos eine Masse, meinte ich eine träge Masse,
    Eine Ruhemasse kann ich aus der Energie immer theoretisch konstruieren. Das ist nicht die Frage. Da kann man sehen, wo mangelnde Prazision hinführt.

  65. @fossilium 2. Juli 2018 @ 23:25

    Quatsch, ich bin doch kein Einsteinleugner.

    Sorry. Wenn dem nicht so ist, nehme ich Sie aus der Aussage raus. Nun noch einige Anmerkungen zu Ihren Fragen:

    …ich habe aber gefragt, wo kommt der Impuls der Strahlung her, …Es ist mir egal, ich will es gar nicht wissen, ich will nur etwas verstehen – und so sehr ich mich anstrenge, es ist ein Punkt, der bei der SRT (von mir jedenfalls) nicht verstanden ist.

    Nun der Impuls der EM-Strahlung wird meines Wissens nicht durch die SRT sondern durch die Quantenphysik erklärt. Aber diese ist nicht besonders anschaulich.

    Ich glaube nicht, eher setzt das Fermatsche Prinzip die Konstanz der Lichtgeschwindigkeit voraus. Es könnte sogar sein, dass es selbst voraussetzt, das die Geschwindigkeit immer über den kürzesten Weg zwischen zwei Punkten definiert ist, und von daher diese eigene Voraussetzung im Zirkelschluß ableitet. Das wäre ja ein Ding, wenn daß bisher keinem (auch mir nicht) aufgefallen ist.

    Nein das stimmt nicht. Das Fermatsche Prinzip erklärt auch das Brechungsgesetz. Denn die Aussage ist ja der zeitlich kürzeste Weg. Vom Wellenmodell des Lichtes ist es auch einleuchtend. Übrigens muss es nicht immer das absolute Minimum sien (Long Path bei der Kurzwelle).

  66. @fossilium

    Nachtrag:

    Es gab schon früher Überlegungen aus der Elektrodynamik, daß elektromagnetische Wellen eine träge Masse bezüglich Beschleunigung haben. Hier eine Anmerkung aus Wikipedia.

  67. Hallo Herr Knoth,

    was soll das sein: der zeitlich kürzeste Weg ?
    Ist das ein anderer als der räumlich kürzeste Weg ?

    Die Lagrangefunktion ist eine Funktion von Ort, Zeit und Geschwindigkeit und die Varianz des Zeitintegrals der Lagrangefunktion ist für den kürzesten Weg null. Wenn natürlich der kürzeste durchlaufene Weg schon in der Definition der Geschwindigkeit drinsteckt, ist dann nicht schon vorausgesetzt, was die Varianz des Zeitintegrals erst zeigen soll, nämlich dass in der Natur der kürzeste Weg durchlaufen wird ?

    Grüße Fossilium

  68. @fossilium heute 10:02

    was soll das sein: der zeitlich kürzeste Weg ?
    Ist das ein anderer als der räumlich kürzeste Weg ?

    Ja das ist richtig. Man kann das Brechungsgesetz aus dem Fermatschen Gesetz ableiten. Der Weg zwischen zwei Punkten ist dann natürlich nicht der räumlich kürzeste Weg. Denn im optisch dünneren Medium ist der Weg länger als im dichteren Medium.

  69. https://de.wikipedia.org/wiki/GPS-Technik#Lösungsverfahren_für_4_Gleichungen_mit_4_Unbekannten

    Dort steht:

    Der GPS-Empfänger befinde sich zur GPS-Systemzeit t_0 an einem Ort mit den Koordinaten x_0 , y_0 , z_0

    Der Ursprung des Koordinatensystems wird wohl im Erdmittelpunkt liegen. Aber wo befinden sich die x,y,z-Achsen? Und rotieren diese Achsen mit der Erde mit?

    Das würde aber meiner Meinung nach keinen Sinn machen, denn

    (x_1-x_0)²+(y_1-y_0)²+(z_1-z_0)²=[c(t_1-t_0)]²

    Der Autofahrer befindet sich zur Zeit t_0 bei (x_0,y_0_z_0)
    Der Satellit befindet sich zur Sendezeit t_0 bei (x_1,y_1_z_1)
    Beim Autofahrer kommt das Signal zur Zeit t_1 an

    Die nichtrotierenden Koordinaten (x_0,y_0_z_0) werden dann wohl in die altbekannten rotierenden Längen und Breitengrade umgerechnet?

    Ist meine Vermutung korrekt oder lieg ich da völlig falsch?

  70. Hi zusammen,
    um das Thema abzuschließen: ich meine das Problem des Impulses muß etwas damit zu tun haben, daß die SRT keine allumfassende Theorie ist, und zwar in dem Sinne, daß Sie nur Aussagen zur Raumzeitgeometrie macht, wegen c = const, und alle materiellen Aspekte außen vor bleiben. Der Einbezug der materiellen Aspekte erfolgt erst in der ART. Also der Imuls hat etwas mit den “materiellen” Aspekten der Strahlung zu tun, und deshalb kann man ihn im Rahmen der SRT nicht schlüssig ableiten oder begründen. Das bedeutet, daß auch das, was Strahlung ist, also alles was sich mit c bewegt, nicht aus der SRT erklären kann. Die SRT beschreibt nur die Folgen von c=const auf die Raumzeit und das Trafo-Verfalten der Bezugssysteme, ihr Schwerpunkt liegt nicht auf Einzelobjekten, sondern auf der Struktur von Raum und Zeit, und dabei ist sie eine vorläufige Theporie, weil sie das, was Raum und Zeit beugt, nicht erklären kann. Dennoch gehört sie zu Recht zu den größten geistigen Errungenschaften der Menschheit, egal ob Einstrein irrte oder nicht. Er irrte aber nicht, wahrscheinlich wußte er genau, wo die “Grenzen” der SRT sind.
    Grüße Fossilium

  71. @Julian Apostata

    Das ist richtig. Es handelt sich um ein Cartesisches Koordinatensystem, mit dem Ursprung im Zentrum, das sich nicht mitdreht. Das macht die Berechnung am einfachsten. Die Koordinaten werden aber gleich in ein geodätisches Koordinatensystem, zum Beispiel World Geodetic System 1984 umgerechnet. Das ist ein mit rotierendes elliptoidisches Koordinatensystem.

  72. @fossilium 3. Juli 2018 @ 15:11

    um das Thema abzuschließen: ich meine das Problem des Impulses muß etwas damit zu tun haben, daß die SRT keine allumfassende Theorie ist, und zwar in dem Sinne, daß Sie nur Aussagen zur Raumzeitgeometrie macht, wegen c = const, und alle materiellen Aspekte außen vor bleiben. Der Einbezug der materiellen Aspekte erfolgt erst in der ART.

    Nun die “materiellen” Eigenschaften wie “träge Masse” bei der Beschleunigung. kann man aus der Elektrodynamik ohne SRT wie aus dem Link im Beitrag heute um 8:04 verstehen. Denn die stehende Welle hat die messbare Masse m=E/c^2. Wenn man eine Seite des Resonators öffnte entweich diese Masse mit der Geschwindigkeit c und hat dann den Impuls p=E/c.

  73. Julian Apostata 4. Juli 2018 @ 12:55

    Nun ich vermute, daß sie damit die Sagnac-Korrektur für die Synchronisation der Uhren mit bekanntem Standort mit den Satelliten meint. Dabei ist dies wenn ich die Formel richtig verstanden habe, der Term -x*v/c**2 allerdings als Skalarprodukt von Richtungsvektor zum Satelliten und der Geschwindigkeit des Satelliten. Möglicherweise liege ich damit falsch?

  74. @Joachim Schulz
    Eins vorweg: Ich will hier nicht die Mathematik der Relativitätstheorie kritisieren, sondern ihre populärwissenschaftliche Erklärung. Die mit Hilfe der Mathematik der RT gemachten Voraussagen wurden durch Beobachtung eindrucksvoll bestätigt.

    Ich halte die Vermengung von Ruhesystemen einerseits und Inertialsystemen andererseits (und allem, was jeweils daran hängt) für problematisch. In vielen Köpfen frisst sich ein Bild fest von einem vereinigten Koordinatensystem mit den Vorteilen von beiden Systemen: man hat da das ganze Universum in einem Koordinatensystem, dessen Koordinaten man selbst zuverlässig messen kann.

    Meiner Ansicht nach ist diese scheinbar mögliche Gleichsetzung nur das Artefakt von extrem speziell gewählten Vorgaben, die man auch Anfängern zur Berechnung überlassen kann. Jenseits dieser Vorgaben existieren in der SRT immer noch globale Inertialsysteme und lokale Ruhesysteme, die aber klar verschiedene Eigenschaften haben.

    Ein Inertialsystem ist dabei ein beobachterunabhängiges globales Koordinatensystem, das die Raumzeit und ihre Inhalte repräsentieren kann. Dabei kann das Inertialsystem nur dann die Raumzeit zuverlässig repräsentieren, wenn es nicht nur Objekte an jedem Ort erlaubt, sondern auch ihre Bewegungen in jede Richtung.

    Außerdem hat auch jeder Beobachter sein eigenes Ruhesystem (wobei “Beobachter” auch ein beliebiges massebehaftetes Objekt sein kann – nur bei Photonen würde es nicht klappen). Und alle Ruhesysteme sind gleichwertig in dem Sinn, dass darin die normalen Naturgesetze erfahren werden, egal wie lange sich ein Beobachter schon in diesem Ruhesystem befindet. Nur befindet sich eben nur der Beobachter in seinem Ruhesystem, zumindest was die Zuverlässigkeit der Beobachtung durch selbst gemessene Daten angeht.

    Um die Auswirkungen von Objekten auf andere Objekte zu bestimmen, müsste man von dem Ruhesystem des ersten Objekts die Auswirkungen nach außen in ein gemeinsames globales Inertialsystem übertragen (und man kann dabei verschiedene wählen, muss aber nach der Auswahl des globalen Systems konsequent bei diesem bleiben), und von diesem globalen System wiederum zurück in das lokale Ruhesystem eines empfangenden Beobachters.

    Wenn man die besonderen Vorgaben hat, dann kann man bei einem Schritt eine Koordinatenumrechnung von 1:1 von gemessenen zu globalen Koordinaten haben – aber das hätte man teuer bezahlt durch den Verlust der Allgemeingültigkeit.

    Aber so richtig fiese Probleme würde man kriegen, wenn man durch diese Gleichsetzung jenseits speziell-relativistischer Probleme wie dem “Zwillingsparadoxon” noch diese Idee in die Betrachtung der Allgemeinen Relativitätstheorie mitnimmt.

  75. @Frank Schmidt 5. Juli 2018 @ 00:32

    Meiner Ansicht nach ist diese scheinbar mögliche Gleichsetzung nur das Artefakt von extrem speziell gewählten Vorgaben, die man auch Anfängern zur Berechnung überlassen kann. Jenseits dieser Vorgaben existieren in der SRT immer noch globale Inertialsysteme und lokale Ruhesysteme, die aber klar verschiedene Eigenschaften haben.

    Nun da habe ich die SRT aber anders verstanden. Und so kommt sie bei meiner Lektüre des Artikels anders rüber.

    Ein Inertialsystem ist dabei ein beobachterunabhängiges globales Koordinatensystem, das die Raumzeit und ihre Inhalte repräsentieren kann. Dabei kann das Inertialsystem nur dann die Raumzeit zuverlässig repräsentieren, wenn es nicht nur Objekte an jedem Ort erlaubt, sondern auch ihre Bewegungen in jede Richtung.

    Also etwas, das von Objekten und Beobachtern unabhängig ist. Soll das ein absoluter Raum sein?

    Außerdem hat auch jeder Beobachter sein eigenes Ruhesystem (wobei “Beobachter” auch ein beliebiges massebehaftetes Objekt sein kann – nur bei Photonen würde es nicht klappen). Und alle Ruhesysteme sind gleichwertig in dem Sinn, dass darin die normalen Naturgesetze erfahren werden, egal wie lange sich ein Beobachter schon in diesem Ruhesystem befindet.

    Aber genau das sind doch die Inertialsysteme. Ein relativ zu einem anderen Objekt sich gleichförmig bewegenden Objekt kann ein Inertialsystem zugeordnet werden, in dem Beobachter/Objekt ruht. Inertialsystem ist nach meinem Wissen ein Koordinatensystem, das sich relativ zu einem anderen Inertialsystem sich unbeschleunigt bewegt.

    Um die Auswirkungen von Objekten auf andere Objekte zu bestimmen, müsste man von dem Ruhesystem des ersten Objekts die Auswirkungen nach außen in ein gemeinsames globales Inertialsystem übertragen (und man kann dabei verschiedene wählen, muss aber nach der Auswahl des globalen Systems konsequent bei diesem bleiben), und von diesem globalen System wiederum zurück in das lokale Ruhesystem eines empfangenden Beobachters.

    Wieso muss man ein globales Inertialsystem benutzen, in dem das Objekt nicht ruht? Wenn zwei Objekte relativ zueinander mit konstanter Geschwindigkeit sich bewegen, dann hat jedes davon sein Inertialsystem. Oder habe ich da was falsch verstanden?

  76. @Frank Schmidt
    Ich fürchte, Sie haben genau die Punkte nicht ganz verstanden, die ich in diesem Artikel erklären wollte:
    Ein Ruhesystem ist nichts anderes als ein (beliebiges) Inertialsystem, in dem das betrachtete Objekt die Geschwindigkeit v=0 hat. Die SRT, wie auch schon die Newtonsche Physik, besagt, dass physikalische Gesetze in jedem Inertialsystem gleich sind. Daher kann man mechanische und elektromagnetische Vorgänge in jedem beliebigen Inertialsystem berechnen und verstehen. Auch in jedem Ruhesystem.

    Welches System zu den einfachsten Formeln führt, ist sehr von der Situation abhängig. Neben den Ruhesystemen kommen Schwerpunktsysteme und Laborsysteme in Frage.

  77. @Joachim Schulz
    OK, vielleicht anders formuliert. Ein Objekt (das momentan nicht beschleunigt und sich in einer flachen Raumzeit befindet) hat ein Inertialsystem (das Ruhesystem), in dem das Objekt die Geschwindigkeit v=0 hat. Dort erfährt es die normalen Naturgesetze. Und alle derartigen Objekte und ihre Ruhesysteme sind gleichberechtigt, und keins ist besonders hervorgehoben.

    Das bezweifle ich doch gar nicht. Ich will darauf hinaus, dass in Fällen, wo es eben doch irgendwann eine Beschleunigung gibt, die am Anfang gewählten (globalen, zuverlässigen) Inertialsysteme erhalten bleiben, während das Objekt dann relativ zu einem anderen Ruhesystem die Geschwindigkeit v=0 hat. Es erfährt in diesem neuen Ruhesystem wiederum die normalen Naturgesetze.

    Aber Zeiten und Entfernungen werden vom Objekt dann anders gemessen als im ursprünglichen Inertialsystem. Das bedeutet nicht, dass das zweite System schlechter ist als das erste. Aber wenn man die Bewegung vieler Objekte zuverlässig erfassen will, braucht man ein Inertialsystem, das man zwar beliebig wählen kann, bei dem man aber bleiben muss. Wenn man dagegen immer das momentane Ruhesystem verwendet, kommt man zu scheinbaren Widersprüchen wie dem Zwillingsparadoxon.

  78. @Frank Schmidt,

    Ja, so formuliert würde ich es auch unterschreiben. Die Schwierigkeit entsteht, wenn der Eindruck erweckt wird, das Ruhesystem sei eine Eigenschaft des Objektes.

  79. @Frank Schmidt 5. Juli 2018 @ 20:04

    Aber wenn man die Bewegung vieler Objekte zuverlässig erfassen will, braucht man ein Inertialsystem, das man zwar beliebig wählen kann, bei dem man aber bleiben muss.

    Genau dies ist im Falle der Mikrowellenzentrifuge von Herrn Thim der Fall. Betrachtet man den ganzen Vorgang aus dem Laborsystem, dann ist klar, daß mein keinen transversalen Dopplereffekt misst.

  80. @Joachim Schulz
    Ich wollte ursprünglich mit meinen Überlegungen zu Schwarzen Löchern in der Allgemeinen Relativitätstheorie weitermachen, aber Sie hätten mich mit der gängigen, leider nur halbwissenschaftlichen Argumentation zurückgewiesen, und Ihnen wäre nicht klar gewesen, worin der Fehler in dieser Argumentation liegt.

    Also habe ich jetzt diese problematische Art der Argumentation auf ein anderes Problem übertragen, Planetenbahnen in der newtonschen Physik. Wäre dort ähnlich halbwissenschaftlich argumentiert worden, gälte das Problem wohl heute als ungelöst oder sogar unlösbar. (Entsprechend wäre das Problem Schwarzer Löcher mit vollständig wissenschaftlicher Argumentation schon vor Jahrzehnten gelöst worden)

    Wie gesagt, es ist nur ein Beispiel, nicht das, was wirklich über Planetenbahnen gesagt wurde. Ich will Ihren Blick dadurch auf problematische Argumentationsweisen und ihre zerstörerische Wirkung richten.

    Schritt 1: Man ist noch sicher im wissenschaftlichen Territorium. Man hat sich entschieden, die newtonsche Physik zu nutzen, und hat deren allgemeine Anwendbarkeit in vielen Experimenten überprüft.

    Jetzt ist man eigentlich ganz nah dran, das Problem zu lösen, man weiß aber noch nicht exakt, was die newtonsche Physik exakt für Planetenbahnen bedeutet.

    Schritt 2: Die schöne Einzellösung. Professor Y berechnet, dass man eine schöne Lösung erhält, wenn man die Sonne und einen Planeten betrachtet. Sie umkreisen einander regelmäßig. Wählt man ein Koordinatensystem mit der Sonne im Zentrum, dann beschreibt der Planet eine wunderschöne Keplerbahn.

    Schritt 3: Der Gewöhnungseffekt (Populärwissenschaft I). Professor Y lehrt seine Lösung schon eine Weile, und es harmoniert auch damit, wie man sich die Welt vorstellt. Man hat gelernt, die Keplerbahnen als “die Realität” zu verstehen.

    Schritt 4: Der handwerkliche Fehler (Populärwissenschaft II). Professor Y zeigt, dass die Keplerorbits von Planeten unabhängig von der Lage der Bahn im Raum, der Entfernung von der Sonne und der Exzentrizität der Bahn ist. Also schließt er, dass man daraus ein allgemeines Koordinatensystem mit der Sonne im Zentrum und Planeten, die sie in Keplerorbits umkreisen, erstellen kann.

    Wenn hier nicht der Aufschrei kommt, oder er zu leise ist und Professor Y weitermachen kann, hat man ein Problem.

    Schritt 5: Das alternative Problem. Professor Y wird natürlich Widerspruch ernten, und hat darauf auch schon eine Erklärung: Wählt man sein Koordinatensystem, hat man schöne Keplerbahnen und, wichtiger noch, orbitale Stabilität. Bestünde man dagegen störrisch darauf, ein Koordinatensystem zu benutzen, in dem sich die Sonne mal hierhin und mal dorthin bewegte, würden die Abstände der Planeten von der Sonne zusätzlich zu ihren Orbits schwanken. Man könnte sie nicht mal langfristig berechnen.

    Man würde natürlich gern in einer Welt mit orbitaler Stabilität leben. Aber das entschuldigt nicht Professor Y’s Verfälschung der newtonschen Theorie, die darum auch ein falsches Bild der Welt liefert. Weil Schritt 4 die Wissenschaft selbst verfälscht, kann auch die wissenschaftliche Untersuchung der orbitalen Stabilität in Schritt 5 den Grundfehler nicht aufheben oder entschuldigen.

    Schritt 6: Die Konstruktion der Echohalle (Populärwissenschaft III). Angenommen, Professor Y wurde nie von seinen Kollegen zurückgepfiffen, dann hat man am Ende folgende Situation: Man geht zu Doktor A und sagt, dies könne so doch nicht stimmen, man solle doch lieber Newton verwenden. Und Doktor A entgegnet: “Glauben Sie mir, ich habe mich lange damit beschäftigt. Im Grunde spräche nichts dagegen, dass die Planeten sich so newtonsch um die Sonne bewegen würden, aber Sie wissen doch, dass ein Sonnensystem auch orbitale Stabilität braucht. Ich habe selbst untersucht, wie sich Orbits in einem solchen newtonschen Fall entwickeln würden, und musste leider feststellen, dass in vielen Fällen Körper auf extreme Bahnen geraten und schließlich aus dem System geschleudert würden. Also liegt dort leider keine Lösung. Aber vielleicht löst ja einer meiner Studenten das Problem der Grenzentfernung, die festlegt, in welchen Fällen ein Körper direkt die Sonne umkreist und in welchen Fällen er stattdessen einen anderen Körper umkreist. Die Technologie entwickelt sich ja laufend, irgendwann wird bestimmt ein Durchbruch gelingen.”

    Nun, was denken Sie über Doktor A und was würden Sie ihm gerne sagen? Er hängt in den Klauen der Populärwissenschaft und ist sich dessen nicht mal bewusst.

    Um zum Hauptgedanken zurückzukehren: Ich denke, die Probleme Schwarzer Löcher haben eine Lösung, und diese Lösung ist Einsteins Allgemeine Relativitätstheorie. Diese ist allgemein gültig, aber die Verfälschung, die sich als Verbesserung ausgibt, ist es nicht und wird zu Unsinn, wenn man sie als allgemein gültig erklärt.

  81. @Joachim Schulz
    Vielleicht fragen Sie wie Markus Pössel: “Geht das nicht kurz und knackig?”

    Nehmen wir Ihren eigenen Satz “Inertialsysteme sind globale Koordinatensysteme”. Das, was ein Inertialsystem in der SRT zur Beschreibung des Universums geeignet macht, ist, dass es ein globales Koordinatensystem ist. Es enthält alle Punkte des Universums und Bewegungen durch diese Punkte in alle Richtungen. Und zwar unabhängig davon, ob ich als Erbauer eines Gedankenexperiments diese nun nutzen will oder nicht, und ob ich dieses Inertialsystem nutzen will oder ein anderes.

    Wenn ich das Zwillingsparadox in einem Koordinatensystem korrekt beschreiben will, muss ich aber ein Inertialsystem verwenden. Die andere Eigenschaft für unbeschleunigte Systeme, dass ein Inertialsystem auch ein Ruhesysten für einen Beobachter sein kann, ändert sich im Laufe des Gedankenexperiments – verwende ich ständig Daten aus den momentanen Ruhesystemen der Zwillinge, erhalte ich den scheinbaren Widerspruch.

    Das sollte eigentlich so selbstverständlich sein, wie sich als Arzt die Hände zu waschen, und ich sollte das nicht zu sagen brauchen. Dann könnte ich einfach sagen: Das akzeptable Koordinatensystem ist ein globales, das die Punkte des Universums und Bewegungen in alle Richtungen beschreibt, und nicht eins, das man nur durch willkürliche Wahl eines Gedankenexperiments erhält, in dem man zwar einen oder viele unbeschleunigte Beobachter hat, das aber nicht die geringste Richtungs- oder Geschwindigkeitsänderung zulässt und so praktisch nichts in Universum beschreibt.

    Ich fürchte, wenn ich mich so kurz fasse, kommt nur der “Beamten-Dreisatz” (Haben wir schon immer so gemacht. Haben wir noch nie so gemacht. Da könnte ja jeder kommen) und nicht die geringste Prüfung, ob das eigene Denkmodell auch mit der Relativitätstheorie übereinstimmt oder ob man das bloß behauptet.

  82. @Frank Schmidt
    Das war kurz und knackig. Vielen Dank. Ich habe eine Weile nicht reagiert, weil ich zwei Wochen Familienurlaub hatte und in der Zeit bewusst kaum Internetverbindung hatte. Manchmal haben andere Priorität.

    Ich glaube, dass das Problem gerade in der SRT nicht so groß ist, wie Sie es sehen. Die Physiker sind erstens ausreichend mit klassischer Mechanik vertraut, um diese Zusammenhänge zu kennen und zweitens spielt die SRT in der Grundlagenforschung eine eher zurückhaltende Rolle. Die Quantenmechanik und ihre Deutungsmuster sind da prominenter. Und wo es um kosmologische Modelle geht, muss man die ART heranziehen.

    Sie haben aber grundsätzlich recht: Es besteht die Gefahr, ausgetretene Wege beizubehalten und dabei Lösungen abseits des bekannten zu übersehen. Viele Deutungsmuster in der Physik sind historisch gewachsen und es gibt erkenntnistheoretisch keine Garantie, dass wir uns auf dem richtigen Weg befinden. Vielleicht braucht es einen Paradigmenwechsel um, voranzukommen. Vielleicht aber auch nicht. Im Grunde muss die Forschung immer beides abdecken: Vorhandene Theorien weiterentwickeln und nach Erweiterungen suchen und neue Wege erkunden.

  83. @Joachim Schulz
    Ich sehe das Problem eigentlich nicht in der SRT, sondern in der Weise, wie die mathematischen Ergebnisse der ART erklärt werden.

    Die Lösung liegt meiner Ansicht nach nicht im Unbekannten abseits der bekannten Pfade, sondern im Gegenteil im Zentrum. Sie wird an allen Universitäten gelehrt und ihre Voraussagen wurden extrem gut durch Beobachtungen bestätigt, was dann auch als Triumph der Allgemeinen Relativitätstheorie gewertet wird, meiner Ansicht nach zu Recht.

    Der nötige Paradigmenwechsel ist meiner Ansicht nach genau der, den die Relativitätstheorie bringen sollte: Weg vom “Als Beobachter messe ich absolute Dinge und verstehe das Universum direkt” und hin zum “Als Beobachter messe ich Dinge relativ zu mir. Aber meine Messergebnisse und die aller anderen Beobachtern können in ein globales Koordinatensystem übertragen werden, mit dessen Hilfe das Universum verständlich wird”.

    Stattdessen wird ein Jahrhundert nach der Relativitätstheorie wieder gepredigt, dass man unbedingt einen Beobachter verwenden müsse, um das Univerum zu verstehen.

    Ich befürchte, es ist so gelaufen, dass Professoren Methoden entwickelt haben, um ihre Studenten schneller an die Mathematik der Relativitätstheorie heranzuführen, und dass diese die Methoden nicht als die Vereinfachungen, Merksätze und Gleichnisse verstanden haben, die sie waren, sondern als Ewige Wahrheiten, und dies an die eigenen Studenten weitergegeben haben. So dass Generation um Generation die Lehre weiter Richtung Vorurteil abzudriften droht, und nur die Experimente, die Wissenschaftler natürlich weiterhin machen und diesen vertrauen, einen Teil dieser Drift abwehren können.

    Aber dort, wo Experimente nicht hinkommen, kann sich der Müll ansammeln. Dort gibt es immer noch deutliche Warnungen durch die inneren Widersprüche, die sich dort ergeben, aber da heißt es, dass das Universum eben so ist, und dies wäre mathematisch bewiesen. Nur ruht dieser “Beweis” auf dem Dogma des absoluten Beobachters, in dessen System angeblich alles Sinn ergibt.

    Aber schon bei einfachen Problemen wie dem “Zwillingsparadox” haben die Erklärer Probleme damit, festzulegen, welcher Beobachter denn der absolute wäre – es lässt sich zu leicht ein Gegenbeispiel konstruieren. Ein beobachterunabhängiges System wie ein Inertialsystem in der SRT würde mit diesen Gegenbeispielen dagegen gut klarkommen.

    Und auch in der ART könnte mit beobachterunabhängigen Systemen, in die die Beobachtungen verschiedener Beobachter transformiert werden, das Universum besser verstanden werden, und die durch Beobachter-Zentriertheit entstandenen inneren Widersprüche würden nicht auftreten.

  84. Fragmentarisches“ zur „Erinnerung“:
    Wenn Euklid (…lebte wahrscheinlich im 3. Jahrhundert v. Chr.) noch nach plausibler Anschauung für mathematische Grundlagen suchte und somit eine interdisziplinäre Verbindung herstellte, die man als richtig oder falsch bewerten konnte, so stellt sich in der modernen Mathematik die Frage nach richtig oder falsch nicht. Euklids Definitionen sind explizit, sie verweisen auf außermathematische Objekte der „reinen Anschauung“ wie Punkte, Linien und Flächen. “Ein Punkt ist, was keine Breite hat. Eine Linie ist breitenlose Länge. Eine Fläche ist, was nur Länge und Breite hat.”

    Als David Hilbert (1862 – 1943) im 20. Jahrhundert erneut die Geometrie axiomatisierte, verwendete er ausschließlich implizite Definitionen. Die Objekte der Geometrie hießen zwar weiterhin „Punkte“ und „Geraden“ doch sie waren lediglich Elemente nicht weiter explizierter Mengen. Angeblich soll Hilbert gesagt haben, dass man jederzeit anstelle von Punkten und Geraden auch von Tischen und Stühlen reden könnte, ohne dass die rein logische Beziehung zwischen diesen Objekten gestört wäre…

    Exemplarisch über Johann Carl Friedrich Gauß (1777 – 1855), Georg Friedrich Bernhard Riemann (1826 – 1866), der u.a. den Begriff der Metrik einführte…welcher modern bedeutet: Eine Metrik ist ein Skalarprodukt auf dem Tangentialraum einer Mannigfaltigkeit, die in differenzierbarer Weise von einem Punkt abhängt. Die euklidische Metrik ist dabei lediglich ein Spezialfall… Geometrie ist im Riemannschen Sinne »innere Geometrie«, deren Objekte Größen sind, die nur von den lokalen Eigenschaften einer Metrik abhängen…
    folgen Felix Christian Klein’s (1849 – 1925) Ausführungen …zu Veränderungen der Lage – Drehungen, Spiegelungen, Verschiebungen, … – diese sind die Wirkung einer Gruppe, die er als „Transformationsgruppe“ bezeichnet. Die geometrischen Objekte sind Invarianten gewisser Gruppen, die auf Mengen operieren. Mit Rückführung der Geometrie auf die Algebra konnte man Geometrien klassifizieren, indem man eine Grundmenge und die darauf operierende Gruppe angab. Zwischen 1870 und 1920 wurden die mathematischen Axiome neu geschrieben (Dedekind, Cantor), es entstanden die Differentialgeometrie (Poincare, Einstein) und neue Algebren mit bisher unerforschten Symmetrien (Tensorkalkül, Lie-Algebra). Die ersten Experimente zur Atom- und Quantenphysik ließen sich gut mit den bis dahin rein abstrakten mathematischen Formalismen verbinden, was erst zur bekannten Symbiose führte, welche sich sodann im Zuge der Begeisterung für mathematische Möglichkeiten letztendlich von der Realphysik abspaltete und diese im Zuge der Standardmodelle beherrschte.

    William Thurston (1946 – 2012) griff die Ideen von Klein auf. In seiner grundlegenden Definition einer Modellgeometrie verbindet er einen topologischen Raum mit der Wirkung einer Lie-Gruppe, welche gewissen Maximalitätsbedingungen genügt. Dieses Werkzeug führte zur vollständigen Charakterisierung aller möglichen Geometrien in der Dimension 3 und zur Klassifizierung aller kompakten, 3-dimensionalen Mannigfaltigkeiten. Lie-Gruppen und Lie-Algebren wurden um 1870 von Sophus Lie (1842 – 1899) in der Lie-Theorie zur Untersuchung von Symmetrien in Differentialgleichungen eingeführt. Unabhängig von Lie entwickelte Wilhelm Killing (1847 – 1923) ähnliche Ideen zum Studium nichteuklidischer Geometrien. Hermann (Klaus Hugo) Weyl (1885 – 1955) veröffentlichte 1913 das Buch Die Idee der Riemannschen Fläche, in dem u.a. das moderne Konzept der Mannigfaltigkeiten erstmals systematisch eingesetzt wurde. In seinem Aufsatz Gravitation und Elektrizität von 1918 führt er erstmals das Konzept einer Eichtheorie ein, zunächst nicht in der heutigen Form, sondern durch einen lokal veränderlichen Skalenfaktor. In Weyl’s Vorlesungen Raum, Zeit, Materie entwickelt er systematisch den Riccischen Tensorkalkül und benutzt die Parallelverschiebung (Levi-Civita) von Vektoren als fundamentalen Begriff. Das Eichprinzip wurde seit seiner Entdeckung lange nur als Nebeneffekt angesehen, der einige Rechnungen vereinfachen kann, aber ansonsten nur wenig Bedeutung hat. Die eigentliche Bedeutung des Eichprinzips wurde erst 1918 von Hermann Weyl erkannt, der mit Hilfe einer Eichtheorie (Invarianz unter Änderung der Längenskala) versuchte, Maxwells Theorie mit der allgemeinen Relativitätstheorie zu vereinen. Dieser Versuch scheiterte, aber Weyl begründete damit eine ganz neue Herangehensweise an physikalische Probleme. Hermann Weyl ist der Begründer der Eichtheorien im heutigen Sinn.

    1954 veröffentlichten Robert L. Mills (1927 – 1999) und Chen Ning Yang (1922 -) eine Arbeit, in der sie die Eichinvarianz der Elektrodynamik verallgemeinerten und dadurch eine Theorie der schwachen und starken Wechselwirkung schufen. In den 1960ern erkannte man, dass alle bisher beobachteten Wechselwirkungen von Elementarteilchen durch Eichtheorien beschrieben werden können. Von da an, musste die Natur der Mathematik genügen.

    Inwieweit jedoch axiomatisch begründete Abstraktionen an realphysikalische Objekte ankoppeln, steht auf einem ganz anderen Blatt. Mathematik schafft keine neuen Erkenntnisse, auch wenn das Theoretische Physiker im Rahmen der Standardmodelle der Kosmologie und Teilchenphysik gerne glauben.

    Für heutige Theoretiker ist die Welt der Mathematik identisch mit dem Erkenntnishorizont des Menschen. Moderner Mathematik wird mit festem Glauben eine konstruktive Komponente zugeschrieben. Diese neuen Glaubensbekenntnisse haben offensichtlich eine enorme Strahlkraft, obwohl die ihr zu Grunde liegenden Ideen allesamt irrational sind. Heutige Experimente zur Erweiterung des Standardmodells der Teilchenphysik verraten schon durch die Namensgebung »Supersymmetrie« welch Geistes Kind ihnen inne wohnt. Experimente sollen Symmetrien, nein müssen Supersymmetrien genügen. Neue zeitinstabile Super-Partner-Teilchen, die nie als solche, so wie ihre Vorgänger, direkt messbar sein werden, kommen eines Tages nach monatelangen Berechnungen mittels Super-Cluster-Rechnern als Teilchenbeschleuniger-Geburten aus vorselektierten Zerfallskanälen…

    In diesen Zusammenhängen sind plakative Fragen… ob Spezielle Relativitätstheorie (SRT) und Allgemeine Relativitätstheorie (ART), Quantenmechanik (QM) und allgemein Quantenfeldtheorie(n) (QFTs) als Denkmodelle “richtig” oder “falsch” sind, irrelevant. Denn die Frage müsste im Einzelfall lauten, inwieweit ein Denkmodell konsistent ist und erkenntnistheoretisch wertvolle Antworten geben kann. Wenn wir über SRT, ART, QM und QFTs “reden”, dann reden wir bei genauer Sicht ausschließlich über mathematische Konzepte und im “Kern” über axiomatische Randbedingungen. Es ist sinnleer ein axiomatisch begründetes Konzept in Frage zu stellen. Die fundamentalen Probleme der Standardmodelle kommen u.a. mit den theoriebedingten Näherungen und Idealisierungen, sowie inkonsistenten Erweiterungen und willkürlichen Zusatzannahmen. Keines dieser Denkmodelle liefert näherungsfreie Lösungen zur Materiebildung. SRT, ART, QM und QFTs sind nicht korrespondierend und auf Grund der postulierten theoretischen Implikationen wortwörtlich unvereinbar. Zur Erinnerung: Auch die Epizykeltheorie als “frühes Standardmodell” scheiterte nicht an der verwendeten Mathematik, die war stets richtig, sondern letztendlich an der phänomenologisch anschaulichen Nichtgeozentrik.

    Um die vielfältigen Zusammenhänge durchschauen und verstehen zu können, bedarf es u.a. beträchtlicher historischer Kenntnisse. Diese gehören leider seit mehreren Jahrzehnten nicht zur Schul- und Universitätsausbildung von Physikern und Mathematikern. Zentrale Fragen einfacher Anschauung werden also weder in einem historischen Kontext erörtert noch im Rahmen der herrschenden mathematischen Modelle adressiert. Es hat sich in der Theoretischen Grundlagen-Physik seit mehr als 100 Jahren eine Denkmodell-Monokultur entwickelt, die den realobjektbefreiten, angewandten mathematischen Formalismus der Quantenfeldtheorien selbst zum physikalischen Phänomen erklärt. In dieser Welt der realobjektbefreiten Wahrscheinlichkeiten und Lagrangedichten führ(t)en auch Schreibfehler zu neuen Lösungen. Übergeordnet scheint die Basisinformation verloren gegangen zu sein, daß selbst ein realobjektfreundliches, plausibles Denkmodell nicht einer objektiven Realität entspricht, sondern stets ein subjektives Abbild “verkörpert”.
    Dirk Freyling

  85. @Dirk Freyling
    Es geht hier nicht um generell nicht fassbare Operationen im rein mathematischen Raum. Es geht um, welche Koordinatentransformationen erlaubt sind, wenn man anhand von Experimenten festgestellt hat, dass die SRT und ART korrekt sind, Wobei man unbedingt weitergehende Annahmen vermeiden sollte, die nicht von Experimenten gedeckt sind und bei Annahme ihrer Korrektheit innere Widersprüche verursachen.

  86. @Frank Schmidt und Interessierte,…
    in der heutigen theoretischen Grundlagenphysik geht es fast ausschließlich um mathematische Konzepte und im Rahmen der RT um Operationen und Basiswechsel in mathematischen Räumen. Ein Problem mathematischen Denkens ist hier u.a. die inhaltlich-reale Unbestimmtheit. In einfachen Worten: Mathematik kann nicht zwischen Staub und Staubsauger unterscheiden.

    Angewandte Realobjekt-Forschung geht von reproduzierbaren, empirischen Befunden aus und bemüht sich dann um Systematisierung, Verallgemeinerung und ein „theoretisches Verständnis“.

    Im heutigen standardmodell-orientierten Denken werden hingegen theoretisch Befunde postuliert, nach denen dann mittels computersimulierten „Versuchsaufbauten“ selektiv gesucht wird. Diese stark theoriebeladene Suche kennt keinen einzigen direkten Nachweis und lässt sich auf Grund der vielen freien Parameter, nicht detektierbaren postulierten Theorieobjekten und deren postulierten Kaskadenereignissen beliebig ergebnis-interpretieren.

    Der Autor dieses Blogbeitrages schreibt u.a.: …“Entscheidend ist die Frage, welche Formen von Koordinatenwechsel die physikalischen Gesetze unverändert lassen…“

    Erst einmal: Welche physikalischen Gesetze? Bei genauer Sicht gibt es keine konsistenten Objektbeschreibungen.

    Beispiel: „Licht“ ist je nach subjektiver Theoriebetrachtung ein nicht weiter phänomenologisch bestimmtes Objekt der Geometrischen Optik, ein Ensemble, beispielsweise im Sinne der mathematischen Beschreibung der Interferenzfähigkeit oder (randbegrenzte) Elektromagnetische Welle oder formal postulierte »mathematische Mischung« der Elektroschwachen Theorie, ein Photon real und/oder virtuell…

    Im Rahmen der SRT besteht die grundsätzliche Aussage-Problematik darin, daß es weder eine Objektphänomenologie gibt, noch – im Sinne physikalischer Interaktionen – real existierende Inertialsysteme.

    Das Verständigungs- und Interpretations-Problem begann bzw. beginnt – wie so oft – mit einem realphysikbefreiten Formalismus. Die beobachtbare Invarianz der (Vakuum-)Lichtgeschwindigkeit ist keineswegs “direkt” mit der Speziellen Relativitätstheorie verbunden, wie oft suggeriert wird. Das historisch viel zitierte Michelson-Morley-Experiment ist eindimensional konzipiert und sagt gar nichts über Masse behaftete Teilchen im Sinne der herrschenden Physik aus und behandelt bei genauer experimenteller Sicht auch keine transversalen Komponenten. Die mathematische Invarianz der transversalen Komponenten ist lediglich eine formale Konsequenz der geradlinig gleichförmig bewegten Beobachter bezüglich kräftefreier Teilchen in einem mathematischen Denkmodell. Mit anderen Worten, daß gesamte Konstrukt der Lorentztransformation(en) ist an Inertialsysteme gebunden. Phänomenologisch sagt die SRT schlicht nichts über die Invarianz der Lichtgeschwindigkeit.

    Bevor hier Mißverständnisse aufkommen, es wird nicht behauptet, das die Mathematik der Relativitätstheorie falsch ist. Wie könnte auch eine axiomatisch begründete These falsch sein? Doch das mathematische Konstrukt besitzt – außer der zu Grunde liegenden beobachtbaren Invarianz der [Vakuum-]Lichtgeschwindigkeit – keine realphysikalische Basis. Es existieren zwar Beobachtungen aber es existiert
    keine Phänomenologie zur Mathematik der SRT. Die inertialsystembehaftete »Relativität der Beobachter« ist eine “heikle Sache”, da es in der Natur nur dann zu messbaren Systemänderungen kommt, wenn Energie ausgetauscht wird. Energieaustausch bedeutet aber grundsätzlich, daß “Kräfte” wirkten oder anders ausgedrückt, daß Beschleunigungen auftraten. Mit der Beschleunigung “verabschiedet” sich das Inertialsystem und folgerichtig gleichfalls die Lorentztransformationen. Die Mathematik der SRT ist nicht falsch sondern schon “per Definition” nicht dynamisch.

    Physik bedeutet Wechselwirkung und Energieaustausch. Koordinatensysteme und Koordinatentransformationen “sind” keine physikalischen Vorgänge, sondern Mathematik. Es wird nicht geklärt, wie die Energiedifferenz und die „Struktur der Energiespeicherung“ über Koordinatentransformationen erzeugt wird oder „verschwindet“.

    Das gesamte gegenwärtige makroskopische physikalische Weltbild baut auf dem Paradigma der „physikalischen Raumzeit“ auf. Die Theorie des geschlossenen sich ausdehnendem Universums und die Urknalltheorie sind das Resultat mathematischer Berechnungen der gedanklich konstruierten Raumzeit.
    Gemäß ART-Postulat trägt nicht nur Masse, sondern auch jede Form von Energie zur Krümmung der postulierten Raumzeit bei. Dies gilt einschließlich der mit der Gravitation selber verbundenen Energie. Daher sind die “einsteinschen” Feldgleichungen nichtlinear. Das grundsätzliche Problem der ART ist ein mathematisches.

    Zur Beliebigkeits-Problematik der freien Parameter des Standardmodells der Kosmologie (Lambda-CDM-Modell) gesellt sich die unvermeidbare „axiomatische Verletzung“ des Kovarianzprinzips. Salopp “formulierter” Hintergrund: Die Allgemeine Relativitätstheorie wurde u.a. aus der Forderung geboren, zur Beschreibung der Naturgesetze beliebige Koordinatensysteme verwenden zu können. Entsprechend dem Kovarianzprinzip sollte die Form der Naturgesetze nicht entscheidend von der Wahl des speziellen Koordinatensystems abhängen. Diese Forderung ist ursächlich mathematisch und führt zu einer Vielfalt von möglichen Koordinatensystemen [Metriken].

    Auch von Relativisten gilt somit unbestritten: Die Gleichungssysteme (Einstein, Friedmann) der Allgemeinen Relativitätstheorie, die den Aussagen des Standardmodells der Kosmologie zu Grunde liegen, liefern keine analytischen Lösungen. Erst Idealisierungen und Näherungen führen begrenzt zu rechenbaren Lösungen. Die unvermeidbaren (“kovarianten”) Widersprüche kommen mit den offensichtlich unzulässigen Idealisierungen und Näherungen des Systems von nichtlinearen, verketteten Differentialgleichungen. Mathematisch kann das Kovarianzprinzip nicht „verletzt“ werden, da es ja axiomatisch begründet ist. Nur diese axiomatische Voraussetzung „entschwindet mit der Verstümmelung“ (Idealisierung und Näherung) der eigentlichen Gleichungen. Mit anderen Worten: Die mathematisch korrekten Gleichungen besitzen keine analytischen Lösungen. Die reduzierten Gleichungen (Näherungen, Idealisierung) besitzen zwar Lösungen, diese sind jedoch nicht kovariant. Somit besitzt keine Lösung eine realphysikalisch begründete Bedeutung. Diese Art des Mathematikgebrauches ist willkürlich, da je nach „Geschmack“ der (selbst)gewählten Metrik andere Ergebnisse erhalten werden.
    Zur Erinnerung und Kenntnisnahme: Die postulierte vierdimensionale “physikalische Raumzeit” wurde nicht von Albert Einstein entwickelt sondern vom Mathematiker Hermann Minkowski (1864-1909). Minkowski hielt am 21.09.1908 in Köln auf der 80. Versammlung der Deutschen Gesellschaft der Naturforscher und Ärzte seinen Vortrag »Raum und Zeit«. In diesem Vortrag führt Minkowski die mathematischen Notationen ein, mit denen die Spezielle Relativitätstheorie Einsteins zur Allgemeinen Relativitätstheorie erweitert werden kann. Aufgrund des Relativitätsprinzips und der daraus folgenden Invarianz gegenüber Lorentztransformationen trägt nicht nur Masse, sondern auch jede Form von Energie zur Krümmung der Raumzeit bei. Dies gilt einschließlich der mit der Gravitation selber verbundenen Energie. Daher sind die einsteinschen Feldgleichungen nichtlinear.

    Unlösbare Probleme der Raumzeit
    Das generelle Problem der Mathematik zur ART ist der Umstand, daß im Rahmen der „Raum-Zeit-Näherungen“ die Theorieelemente der ART letztendlich in einen Newtonschen Raum-Container eingebettet werden. Man formalisiert zwar erst einmal allgemeinrelativistisch aber generiert dann – mittels Idealisierungen und Näherungen – Objekte, wie beispielsweise Schwarzschildmetrik basierende Schwarze Löcher oder eben genäherte Gravitationswellen, und baut diese dann als lokal entstandene Ereignisse in den flachen zeitentkoppelten, absoluten Raum ein. Mit anderen Worten: Die letztendlich zur Anwendung kommende Theorie ist nicht mehr „allgemeinrelativistisch“ im Sinne der ursächlichen Axiomatik und Postulate.

    Ergebnis: Wir können das Geschehen im postuliert allgemeinrelativistischen Kosmos zeitlich nicht als Momentaufnahme festhalten, weil Zeit nicht in der Raumzeit existiert. Es gibt gemäß ART nur die sinnlich und meßtechnisch nicht erfahrbare Raumzeit. Realphysikalische Messungen sind grundsätzlich Raum-Zeit-entkoppelt! Wir messen eine Zeitdifferenz in einem (maximal) dreidimensionalen Raum.
    Das Konzept eines expandierenden Universums, wie wir es heute kennen, war von dem russischen Wissenschaftler Alexander Friedmann und Georges Edouard Lemaitre unabhängig voneinander entwickelt worden. Die postulierte Expansion des Universums darf nicht so verstanden werden, dass sich Galaxien in der Raumzeit voneinander entfernen (Relativbewegung). Es ist die Raumzeit selbst, die sich ausdehnt, die Galaxien werden mitbewegt. Gravitativ gebundene Objekte, wie Galaxien oder Galaxienhaufen, expandieren nicht, denn sie sind durch ihre Eigengravitation von der allgemeinen Expansionsbewegung entkoppelt…

    Verbreitungsstrategie von Objekt- und Entstehungsmythen
    Die menschliche Beobachtungs-Zeitspanne ist verglichen mit den Zeitspannen, in denen sich kosmische Bewegungen abspielten und abspielen, extrem klein. Mit den Angaben aus der menschlichen Beobachtungsdauer Annahmen zu begründen, ist „weit hergeholt“ um es mal salopp zu formulieren. Alle derzeitigen vermeintlich empirischen Messungen sind stark (Urknall-)theoriebeladen. Postulierte Zeitspannen, Entfernungen und Energiedichten sind subjektiv-theorieabhängig.

    Bei allen kosmologischen Beobachtungen handelt es sich nicht um kontrollierbare Laborexperimente. Sowohl in der Newtonschen Interpretation der Gravitation als auch in den „ART-üblichen“ differentialgeometrischen Betrachtungen wird stark idealisiert. Die wahren energetischen Verhältnisse (lokal und teils in dynamischer Überlagerung) lassen sich im wahrsten Sinne der Worte „aus Sicht“ der Erde oder erdnaher Satelliten nicht ermitteln, wobei selbst die von der Erde weit entfernten Satelliten für kosmische Verhältnisse als erdnah zu verstehen sind. Das bedeutet, die Ursachen könnten gänzlich anderer Natur sein als vermutet. Mit steigender Beobachtungskomplexität, die heutzutage durch iterative Nachbearbeitungen stark „(nach-)belastet“ ist, können vermeintliche Beobachtungs-Effekte von mathematischen Artefakten der genäherten „Lösungen“ nicht separiert werden. Schlicht formuliert, man sieht das, was man theoretisch sehen möchte.
    Es beginnt grundsätzlich „ordentlich“, siehe exemplarisch das youtube-video Simulation of the neutron star coalescence GW170817 (https://www.youtube.com/watch?v=V6cm-0bwJ98) Die Beschreibung seitens des Max-Planck-Instituts für Gravitationsphysik (Albert-Einstein-Institut) beginnt mit …“The video shows a numerical simulation“…
    Doch keiner der Verkünder, ob Wissenschaftler, Wissenschaftsjournalist, Nachrichtensprecher, …, „meint“ letztendlich, daß es sich, sowohl theoretisch als auch physisch, um nichts weiter als Hypothesen handelt. Stark theoriebeladene Wünsche werden „im guten (doppeldeutig) materiellen Glauben“ materialisiert. Obwohl jeder sehen könnte, was er nie wirklich sehen wird…

    Wahrnehmungsmöglichkeiten
    In unserem Sonnensystem gibt es weder Neutronensterne, Gamma Ray Bursts (GRBs) noch Schwarze Löcher (respektive „Anomalien“, die als solche interpretiert werden können).
    Eine Liste postuliert „erdnächster“ Schwarzer-Löcher-Kandidaten findet sich unter https://en.wikipedia.org/wiki/List_of_nearest_black_holes mit einer „kürzesten“ Entfernung von 2800 Lichtjahren. Zum Vergleich: Der nächste Stern „aus unserer Sicht“ ist mit 4,24 Lichtjahren Proxima Centauri (https://de.wikipedia.org/wiki/Proxima_Centauri). Objekt- und Entfernungs-Angaben beziehen sich auf die „Sicht des ΛCDM-Modells“.

    Das angesiedelte soziologische Wahrnehmungsproblem „besteht“ darin, daß hier, nach einfachem psychologischem Glaubens-Muster, diverse postulierte Theorieobjekte unterschiedlichster Art, teils seit Jahrzehnten – der mit rudimentärem Wissen ausgestatteten Bevölkerung – als 100% real existent sprichwörtlich “verkauft” werden. Diese fördermittelbringenden Theorieobjekte sollen dann beobachtbare „Kaskadenereignisse“ bilden.
    Es gab mal eine Zeit, da glaubten die meinungsbildenden „Wissensführer“ die Erde sei Mittelpunkt des Universums. Es gab mal eine Zeit, da glaubten die „Wissensführer“ an die Phlogistontheorie. Man kann diesen Menschen nicht nachsagen, daß sie dumm waren. Es gab aus damaliger Sicht vermeintlich gute Argumente, und im Falle der Geozentrik eine begleitende Mathematik, die diese Ansicht unterstützte.
    Im Fall des Lichtäthers erhielt man ein Trägermedium, das Licht leitet, im Fall des Dirac-Sees erhielt man ein Medium im Vakuum, das voll von Energieteilchen ist, die die Fluktuationen im Vakuum verursachen. In beiden Fällen lässt sich das postulierte Medium weder begründen noch nachweisen. “Lustigerweise” lehnen Standardmodell-Anhänger den nicht beobachteten Lichtäther ab und führen ohne mit der Wimper zu zucken sodann die gleichfalls nicht nachweisbare Dunkle Energie und Dunkle Materie sowie ein nicht beobachtbares Higgs-Feld ein. Ausgehend vom Quantenfeldvakuum besteht eine der wortwörtlich großen Schwierigkeiten darin, daß eine Energie im Vakuum angenommen werden muss, die ihre Auswirkungen in die ART „verschleppt“. Es stellt sich in Verbindung mit der kosmologischen Konstante die Frage: Ist die Nullpunktenergie real? Oder verflüchtigt sie sich eines Tages, so wie zuvor Lichtäther und Dirac-See. Nüchtern betrachtet werden die einen Glaubenskonzepte durch andere Glaubenskonzepte ersetzt.

    Dirk Freyling

  87. Was? Beiträge gelöscht? Zensur? VERDÄCHTIG!
    Also noch mal:
    Ein Inertialsystem ist kein globales Koordinatensystem. Der Inertialraum, welcher bei Wikipedia ebenfalls als Inertialsystem gilt, beinhaltet allenfalls unzählige unterschiedlich ausgerichtete Inertialsysteme, weswegen er selbst eben keines mehr darstellt. Wieviele kosmologische Objeke bewegen sich Ihrer Meinung nach gleichförmig, geradlinig und kräftefrei gegenüber einem beliebig gewählten Punkt im Inertialraum? Wer im Inertialraum sein Inertialsystem wechselt, bleibt trotzdem noch im selben Inertialraum.
    https://scilogs.spektrum.de/relativ-einfach/gleichzeitigkeit-ist-relativ/#comment-32176

  88. Nennen Sie es meinetwegen Zensur. Ich werde auch weitere Beiträge dieser Art von Ihnen löschen. Was ein Inertialsystem ist, ist keine frage der Meinung. Es ist klar definiert. Und wenn Sie, wie sie früher einmal zugegeben haben, gar nicht wissen, was ein Inertialsystem ist, dann halten Sie sich bitte einfach zurück.

    Ein Inertialsystem ist ein globales Koordinatensystem mit bestimmten Eigenschaften. So ist es nun einmal in der klassischen Mechanik definiert.

  89. @Joachim Schulz:

    Sie, wie sie früher einmal zugegeben haben, gar nicht wissen, was ein Inertialsystem ist, dann halten Sie sich bitte einfach zurück.

    Ich habe soetwas nie zugegeben, sondern mehr dezent darauf hingewiesen, dass Sie das nicht zu wissen scheinen.

    Ein Inertialsystem ist ein globales Koordinatensystem mit bestimmten Eigenschaften. So ist es nun einmal in der klassischen Mechanik definiert.

    Zwei Objekte bewegen sich gegenüber einem Beobachter nicht geradlinig zueinander. Das ist nun mal global mehr als nur ein Inertialsystem. Aber wie gesagt, Sie sind der Doktor. 😆

  90. Herr Schulz
    Hier darf ich doch darlegen, dass sowohl im Einsteins Gedankenexperimentes als auch im Minkowski Raumzeit absolut unmöglich ist einer der beiden Systeme ( Partikeln, Beobachter ) als ruhend zu betrachten.
    ——————–
    Zarathustra
    7. August 2018 @ 13:13
    Re: Blogs und Foren
    Beitragvon Zarathustra » Di 7. Aug 2018, 10:56

    https://scilogs.spektrum.de/quantenwelt … sicherung/
    Zarathustra
    7. August 2018 @ 10:50
    ————–

    Joachim Schulz
    Zitat:
    Ja, das ist richtig. Als Einstein 1905 “Zur Elektrodynamik bewegter Körper” veröffentlichte, war Peer Review noch nicht üblich. Die Radaktion entschied weitgehend allein, was sie für veröffentlichungswürdig hielten.

    Die Redaktion hat anscheinend den Artikel überhaupt nicht gelesen und die anderen haben wie die gläubigen Menschen daran geglaubt, sonst wäre diese Theorie nicht veröffentlicht oder sofort nach der Veröffentlichung entsorgt.

    Zwei von vielen Gründen, die mit absoluter Sicherheit die obige Feststellung beweisen.
    Das Ziel des Einsteins Gedankenexperiment ist zu zeigen, dass

    a-Die Gleichzeitig sei relativ.
    b– Eine ruhende Uhr tickt schneller als eine bewegte Uhr.

    Zu a-
    Um die Gleichzeitigkeit oder Nicht-gleichzeitigkeit der Ereignisse festzustellen, müssen mindestens zwei Ereignisse Gegenstand der Betrachtung sein, zb. E1 (X1, t 17) und E 2 (X2, t 17).

    Im Einsteins Gedankenexperiment (Arbeit von 1905) gibt es jedoch keine zwei Ereignisse, die im ruhenden (bewegten) System gleichzeitig geschehen, um zu untersuchen, ob die zwei gleichzeitige Ereignisse im ruhenden (bewegten ) System auch vom bewegten (ruhenden) System aus betrachtet als gleichzeitig bewertet werden können oder nicht.
    Da gibt es nichts zu diskutieren. Wirklich absolut nichts. Wenn keine zwei Ereignisse angegeben sind, dann kann nur ein geistig gestörter Mensch über deren Gleichzeitig zu sprechen.

    Zu b-
    Weder die Konstruktion des Gedankenexperimentes noch die mathematische Struktur der Lorentz-Transformation im Minkowski-Raum ermöglichen einer der beiden Systeme (Uhren) als ruhend betrachten zu können.
    Das ist absolut unmöglich. Da gibt es nichts zu diskutieren.

    Auf weitere Argumente, die zeigen, dass diese Theorie bis auf E=mc², von Grund auf falsch ist und zum Teil purer Wahnsinn im klinischen Sinne darstellt.

    PS: Wie oft soll ich wiederholen. Ich bin der letzte Physiklehrer der Menschheit und habe alle wesentlichen offenen Fragen der Physik beantwortet bzw. beantworten kann.
    Wenn ihr wirklich Physik lernen wollt oder an Physik interessiert seid und das alles hier nicht ..(zensiert) ist, dann stellt eure Fragen. Stellt eure Fragen. Ich werde ALLE offenen Fragen der Physik beantworten.

    ##################################################
    Kurze Erläuterung zu a.

    a-Die Gleichzeitigkeit sei relativ.
    b- Eine ruhende Uhr tickt schneller als eine bewegte Uhr.

    Denkblockade oder blinder Glaube an Autoritäten.

    Haben Sie jetzt bemerkt, was euer logisches Denken blockiert? Ich kann nur Vermutungen anstellen, worauf ich jetzt verzichten möchte.

    Denn selbst ein Kind oder ein Erwachsener außerhalb der Anstalt weiß, dass man mindestens zwei Ereignisse (E1 und E2) braucht, die im irgendeinem System gleichzeitig geschehen sind, um untersuchen zu können, ob diese Ereignisse auch von einem anderen System aus betrachtet auch als gleichzeitig bewertet werden kann oder nicht.

    Zu b.
    Im nächsten Beitrag.
    ——————————-
    beendet das größte Verbrechen der Geschichte. Löscht nicht meine Beiträge.
    stellt eure Fragen.
    Stellt eure Fragen.
    Was wollt ihr denn in der Öffentlichkeit. Ich habe die Antworten vorgelegt, wonach die Wissenschaft seit Jahrtausenden sucht.
    Stellt eure Fragen oder beschäftigt euch mit etwas anderem.
    Warum Physik? warum? Warum? wenn ihr nicht mal wissen wollt, was Energie bedeutet, was Masse bedeutet, was …..

  91. @Joachim Schulz
    Ich würde gern herausfinden, in welchen Punkten Ihre und meine Ansichten zu Schwarzen Löchern noch übereinstimmen, und wo sie sich trennen. (Wenn ich diesen Punkt genau wüsste, könnte ich dort genauer untersuchen)

    Zuerst würde ich gerne wissen, ob Ihre Position folgende Aussagen beinhaltet:
    a) Ein frei fallender Beobachter erreicht den Schwarzschildradius nach endlicher Eigenzeit.
    b) Wenn ein relativ zu dem Schwarzen Loch statischer Beobachter dem fallenden Beobachter Signale schickt (mit einem Zeitsignal seiner Eigenzeit), dann empfängt der fallende Beobachter vor Erreichen des Schwarzschildradius nur eine begrenzte Menge mit einem Grenzwert der Zeitsignal-Zeit, die er dort sehen könnte. (Und nicht etwa Signale aus der ganzen verbleibenden Zukunft)
    c) Mit b) verbunden: Wenn selbst Licht mit einem bestimmten Abstand fallende Materie nicht erreichen kann, gilt das umso mehr für nachfolgende fallende Materie, weil diese langsamer als Licht ist. So gibt es nie ein Aufprallsignal der fallenden Materie (anders als bei Neutronensternen), und genau dies wird beobachtet.

    Bitte korrigieren Sie diese Statements, wenn sie Ihnen falsch erscheinen. Und bitte versuchen Sie keine lange abschließende Klärung – da würden Sie wieder an dem Punkt vorbeirasen, an den sich unsere Ansichten trennen, und ich würde wieder nicht erfahren, wo er liegt.

  92. @Frank Schmidt / 12. August 2018 @ 12:20

    Nur eine kurze Anmerkung zu Ihrem b), wenn’s genehm ist.

    Eine fallende Uhr (Beobachter) empfängt tatsächlich alle (d.h. unendlich viele) von der stationären Uhr gesendeten Zeitsignale in der nach ihrer Eigenzeit nur endlich bemessenen Zeitspanne bis zum Erreichen des (scheinbaren) Horizontes.

    Doch das liegt keinesfalls daran, dass die fallende Uhr dabei irgendwie zunehmend “verlangsamt ginge”, wie e.g. der Herr Prof. Rindler es in seinem Lehrbuch suggeriert (der Herr Professor ist halt machmal schon a bisserl desorientiert). Das passende Stichwort hierzu ist indessen gravitational frequency shift.

  93. @Joachim Schulz
    Lesen Sie auch, was Chrys schreibt? Wie denken Sie darüber?

    @Chrys
    Wie erklären Sie dann das fehlende Aufprallsignal? Und wie stehen Sie zu den Messungen im Sonnensystem zu gravitationeller und Bewegungs-Zeitdilatation? Nach denen wäre ein auf eine Masse zufallender Beobachter beiden Formen der Zeitdilatation unterworfen. Oder stehen Sie auf dem Standpunkt, nur das Koordinatensystem des fallenden Beobachters (in dem alles so schön vertraut aussieht) sei wahr, alle anderen dagegen falsch?

  94. Es gibt nur eine Zeitdilatation, man kann sie auf zwei Wegen beschreiben, aber nur einmal benutzen.
    Man darf nicht SRT und Gravitation getrennt nehmen, dann muß man in die ART gucken.

  95. @h 16. August 2018 @ 15:50

    Es gibt nur eine Zeitdilatation, man kann sie auf zwei Wegen beschreiben, aber nur einmal benutzen.
    Man darf nicht SRT und Gravitation getrennt nehmen, dann muß man in die ART gucken.

    Wie ist dann dieser Artikel dann zu verstehen?

    Im Abstract steht folgendes:

    From data up to and including pericentre, we robustly detect the combined gravitational redshift and relativistic transverse Doppler effect for S2 of z = Δλ / λ ≈ 200 km s−1/c with different statistical analysis methods.

    Nun in Zeit-Online etc wurde dann nur vom gravitativen Dopplereffekt geschrieben.

  96. @Frank Schmidt / 15. August 2018 @ 22:04

    Es ereignet sich da kein Aufprall. In der Beschreibung des stationären Beobachters, die so zulässig ist wie jede andere auch, erreicht ein fallendes Testpartikel zu keiner endlichen Zeit den Ort des Horizontes — also nie! Der Horizont selbst ist kein Teil der Schwarzschildschen Lösung, er repräsentiert lediglich eine Grenze für ein sphärisch-symmetrisches Vakuum maximaler Ausdehnung. (Das ominöse “Loch”, dass gern an die Rückseite des Horizontes geklebt wird, ist nicht sphärisch-symmetrisch.)

    Für den fallenden Beobachter wäre der Horizont dadurch charakterisiert, dass die dort empfangenen Zeitsignale mit dem aktuellen Wert +∞ blau-verschoben sein müssten, was jedoch keine sinnvolle Aussage ist. Spätestens jetzt sollte einem klar werden, dass ein durch einen Horizont begrenztes Schwarzschild Vakuum zunächst nur eine unter rein geometr. Annahmen erhaltene Lösung der Feldgl. ist, die zudem von einem noch unspezifizierten Parameter abhängt. Ein physikal. brauchbares Modell wird das erst, wenn man hier tatsächlich eine sphärisch-symmetrische Zentralmasse einführt, die dann durch eine reguläre Randbedingung den noch freien Parameter fixiert, sodass auf diese Weise das von der zentralen Masse generierte Gravitationsfeld in deren Umgebung modelliert wird.

Schreibe einen Kommentar